PrepScholar

Choose Your Test

Sat / act prep online guides and tips, how to write a perfect synthesis essay for the ap language exam.

author image

Advanced Placement (AP)

body-pencil-sharpen-notebook-1

If you're planning to take the AP Language (or AP Lang) exam , you might already know that 55% of your overall exam score will be based on three essays. The first of the three essays you'll have to write on the AP Language exam is called the "synthesis essay." If you want to earn full points on this portion of the AP Lang Exam, you need to know what a synthesis essay is and what skills are assessed by the AP Lang synthesis essay.

In this article, we'll explain the different aspects of the AP Lang synthesis essay, including what skills you need to demonstrate in your synthesis essay response in order to achieve a good score. We'll also give you a full breakdown of a real AP Lang Synthesis Essay prompt, provide an analysis of an AP Lang synthesis essay example, and give you four tips for how to write a synthesis essay.

Let's get started by taking a closer look at how the AP Lang synthesis essay works!

Synthesis Essay AP Lang: What It Is and How It Works

The AP Lang synthesis essay is the first of three essays included in the Free Response section of the AP Lang exam.

The AP Lang synthesis essay portion of the Free Response section lasts for one hour total . This hour consists of a recommended 15 minute reading period and a 40 minute writing period. Keep in mind that these time allotments are merely recommendations, and that exam takers can parse out the allotted 60 minutes to complete the synthesis essay however they choose.

Now, here's what the structure of the AP Lang synthesis essay looks like. The exam presents six to seven sources that are organized around a specific topic (like alternative energy or eminent domain, which are both past synthesis exam topics).

Of these six to seven sources, at least two are visual , including at least one quantitative source (like a graph or pie chart, for example). The remaining four to five sources are print text-based, and each one contains approximately 500 words.

In addition to six to seven sources, the AP Lang exam provides a written prompt that consists of three paragraphs. The prompt will briefly explain the essay topic, then present a claim that students will respond to in an essay that synthesizes material from at least three of the sources provided.

Here's an example prompt provided by the College Board:

Directions : The following prompt is based on the accompanying six sources.

This question requires you to integrate a variety of sources into a coherent, well-written essay. Refer to the sources to support your position; avoid mere paraphrase or summary. Your argument should be central; the sources should support this argument .

Remember to attribute both direct and indirect citations.

Introduction

Television has been influential in United States presidential elections since the 1960's. But just what is this influence, and how has it affected who is elected? Has it made elections fairer and more accessible, or has it moved candidates from pursuing issues to pursuing image?

Read the following sources (including any introductory information) carefully. Then, in an essay that synthesizes at least three of the sources for support, take a position that defends, challenges, or qualifies the claim that television has had a positive impact on presidential elections.

Refer to the sources as Source A, Source B, etc.; titles are included for your convenience.

Source A (Campbell) Source B (Hart and Triece) Source C (Menand) Source D (Chart) Source E (Ranney) Source F (Koppel)

Like we mentioned earlier, this prompt gives you a topic — which it briefly explains — then asks you to take a position. In this case, you'll have to choose a stance on whether television has positively or negatively affected U.S. elections. You're also given six sources to evaluate and use in your response. Now that you have everything you need, now your job is to write an amazing synthesis essay.

But what does "synthesize" mean, exactly? According to the CollegeBoard, when an essay prompt asks you to synthesize, it means that you should "combine different perspectives from sources to form a support of a coherent position" in writing. In other words, a synthesis essay asks you to state your claim on a topic, then highlight the relationships between several sources that support your claim on that topic. Additionally, you'll need to cite specific evidence from your sources to prove your point.

The synthesis essay counts for six of the total points on the AP Lang exam . Students can receive 0-1 points for writing a thesis statement in the essay, 0-4 based on incorporation of evidence and commentary, and 0-1 points based on sophistication of thought and demonstrated complex understanding of the topic.

You'll be evaluated based on how effectively you do the following in your AP Lang synthesis essay:

Write a thesis that responds to the exam prompt with a defensible position

Provide specific evidence that to support all claims in your line of reasoning from at least three of the sources provided, and clearly and consistently explain how the evidence you include supports your line of reasoning

Demonstrate sophistication of thought by either crafting a thoughtful argument, situating the argument in a broader context, explaining the limitations of an argument

Make rhetorical choices that strengthen your argument and/or employ a vivid and persuasive style throughout your essay.

If your synthesis essay meets the criteria above, then there's a good chance you'll score well on this portion of the AP Lang exam!

If you're looking for even more information on scoring, the College Board has posted the AP Lang Free Response grading rubric on its website. ( You can find it here. ) We recommend taking a close look at it since it includes additional details about the synthesis essay scoring.

body-chisel-break-apart

Don't be intimidated...we're going to teach you how to break down even the hardest AP synthesis essay prompt.

Full Breakdown of a Real AP Lang Synthesis Essay Prompt

In this section, we'll teach you how to analyze and respond to a synthesis essay prompt in five easy steps, including suggested time frames for each step of the process.

Step 1: Analyze the Prompt

The very first thing to do when the clock starts running is read and analyze the prompt. To demonstrate how to do this, we'll look at the sample AP Lang synthesis essay prompt below. This prompt comes straight from the 2018 AP Lang exam:

Eminent domain is the power governments have to acquire property from private owners for public use. The rationale behind eminent domain is that governments have greater legal authority over lands within their dominion than do private owners. Eminent domain has been instituted in one way or another throughout the world for hundreds of years.

Carefully read the following six sources, including the introductory information for each source. Then synthesize material from at least three of the sources and incorporate it into a coherent, well-developed essay that defends, challenges, or qualifies the notion that eminent domain is productive and beneficial.

Your argument should be the focus of your essay. Use the sources to develop your argument and explain the reasoning for it. Avoid merely summarizing the sources. Indicate clearly which sources you are drawing from, whether through direct quotation, paraphrase, or summary. You may cite the sources as Source A, Source B, etc., or by using the descriptions in parentheses.

On first read, you might be nervous about how to answer this prompt...especially if you don't know what eminent domain is! But if you break the prompt down into chunks, you'll be able to figure out what the prompt is asking you to do in no time flat.

To get a full understanding of what this prompt wants you to do, you need to identify the most important details in this prompt, paragraph by paragraph. Here's what each paragraph is asking you to do:

  • Paragraph 1: The prompt presents and briefly explains the topic that you'll be writing your synthesis essay about. That topic is the concept of eminent domain.
  • Paragraph 2: The prompt presents a specific claim about the concept of eminent domain in this paragraph: Eminent domain is productive and beneficial. This paragraph instructs you to decide whether you want to defend, challenge, or qualify that claim in your synthesis essay , and use material from at least three of the sources provided in order to do so.
  • Paragraph 3: In the last paragraph of the prompt, the exam gives you clear instructions about how to approach writing your synthesis essay . First, make your argument the focus of the essay. Second, use material from at least three of the sources to develop and explain your argument. Third, provide commentary on the material you include, and provide proper citations when you incorporate quotations, paraphrases, or summaries from the sources provided.

So basically, you'll have to agree with, disagree with, or qualify the claim stated in the prompt, then use at least three sources substantiate your answer. Since you probably don't know much about eminent domain, you'll probably decide on your position after you read the provided sources.

To make good use of your time on the exam, you should spend around 2 minutes reading the prompt and making note of what it's asking you to do. That will leave you plenty of time to read the sources provided, which is the next step to writing a synthesis essay.

Step 2: Read the Sources Carefully

After you closely read the prompt and make note of the most important details, you need to read all of the sources provided. It's tempting to skip one or two sources to save time--but we recommend you don't do this. That's because you'll need a thorough understanding of the topic before you can accurately address the prompt!

For the sample exam prompt included above, there are six sources provided. We're not going to include all of the sources in this article, but you can view the six sources from this question on the 2018 AP Lang exam here . The sources include five print-text sources and one visual source, which is a cartoon.

As you read the sources, it's important to read quickly and carefully. Don't rush! Keep your pencil in hand to quickly mark important passages that you might want to use as evidence in your synthesis. While you're reading the sources and marking passages, you want to think about how the information you're reading influences your stance on the issue (in this case, eminent domain).

When you finish reading, take a few seconds to summarize, in a phrase or sentence, whether the source defends, challenges, or qualifies whether eminent domain is beneficial (which is the claim in the prompt) . Though it might not feel like you have time for this, it's important to give yourself these notes about each source so you know how you can use each one as evidence in your essay.

Here's what we mean: say you want to challenge the idea that eminent domain is useful. If you've jotted down notes about each source and what it's saying, it will be easier for you to pull the relevant information into your outline and your essay.

So how much time should you spend reading the provided sources? The AP Lang exam recommends taking 15 minutes to read the sources . If you spend around two of those minutes reading and breaking down the essay prompt, it makes sense to spend the remaining 13 minutes reading and annotating the sources.

If you finish reading and annotating early, you can always move on to drafting your synthesis essay. But make sure you're taking your time and reading carefully! It's better to use a little extra time reading and understanding the sources now so that you don't have to go back and re-read the sources later.

body-weightlifting-lift-strong

A strong thesis will do a lot of heavy lifting in your essay. (See what we did there?)

Step 3: Write a Strong Thesis Statement

After you've analyzed the prompt and thoroughly read the sources, the next thing you need to do in order to write a good synthesis essay is write a strong thesis statement .

The great news about writing a thesis statement for this synthesis essay is that you have all the tools you need to do it at your fingertips. All you have to do in order to write your thesis statement is decide what your stance is in relationship to the topic provided.

In the example prompt provided earlier, you're essentially given three choices for how to frame your thesis statement: you can either defend, challenge, or qualify a claim that's been provided by the prompt, that eminent domain is productive and beneficial . Here's what that means for each option:

If you choose to defend the claim, your job will be to prove that the claim is correct . In this case, you'll have to show that eminent domain is a good thing.

If you choose to challenge the claim, you'll argue that the claim is incorrect. In other words, you'll argue that eminent domain isn't productive or beneficial.

If you choose to qualify, that means you'll agree with part of the claim, but disagree with another part of the claim. For instance, you may argue that eminent domain can be a productive tool for governments, but it's not beneficial for property owners. Or maybe you argue that eminent domain is useful in certain circumstances, but not in others.

When you decide whether you want your synthesis essay to defend, challenge, or qualify that claim, you need to convey that stance clearly in your thesis statement. You want to avoid simply restating the claim provided in the prompt, summarizing the issue without making a coherent claim, or writing a thesis that doesn't respond to the prompt.

Here's an example of a thesis statement that received full points on the eminent domain synthesis essay:

Although eminent domain can be misused to benefit private interests at the expense of citizens, it is a vital tool of any government that intends to have any influence on the land it governs beyond that of written law.

This thesis statement received full points because it states a defensible position and establishes a line of reasoning on the issue of eminent domain. It states the author's position (that some parts of eminent domain are good, but others are bad), then goes on to explain why the author thinks that (it's good because it allows the government to do its job, but it's bad because the government can misuse its power.)

Because this example thesis statement states a defensible position and establishes a line of reasoning, it can be elaborated upon in the body of the essay through sub-claims, supporting evidence, and commentary. And a solid argument is key to getting a six on your synthesis essay for AP Lang!

Looking for help studying for your AP exam?

Our one-on-one online AP tutoring services can help you prepare for your AP exams. Get matched with a top tutor who got a high score on the exam you're studying for!

Get a 5 On Your AP Exam

Step 4: Create a Bare-Bones Essay Outline

Once you've got your thesis statement drafted, you have the foundation you need to develop a bare bones outline for your synthesis essay. Developing an outline might seem like it's a waste of your precious time, but if you develop your outline well, it will actually save you time when you start writing your essay.

With that in mind, we recommend spending 5 to 10 minutes outlining your synthesis essay . If you use a bare-bones outline like the one below, labeling each piece of content that you need to include in your essay draft, you should be able to develop out the most important pieces of the synthesis before you even draft the actual essay.

To help you see how this can work on test day, we've created a sample outline for you. You can even memorize this outline to help you out on test day! In the outline below, you'll find places to fill in a thesis statement, body paragraph topic sentences, evidence from the sources provided, and commentary :

  • Present the context surrounding the essay topic in a couple of sentences (this is a good place to use what you learned about the major opinions or controversies about the topic from reading your sources).
  • Write a straightforward, clear, and concise thesis statement that presents your stance on the topic
  • Topic sentence presenting first supporting point or claim
  • Evidence #1
  • Commentary on Evidence #1
  • Evidence #2 (if needed)
  • Commentary on Evidence #2 (if needed)
  • Topic sentence presenting second supporting point or claim
  • Topic sentence presenting three supporting point or claim
  • Sums up the main line of reasoning that you developed and defended throughout the essay
  • Reiterates the thesis statement

Taking the time to develop these crucial pieces of the synthesis in a bare-bones outline will give you a map for your final essay. Once you have a map, writing the essay will be much easier.

Step 5: Draft Your Essay Response

The great thing about taking a few minutes to develop an outline is that you can develop it out into your essay draft. After you take about 5 to 10 minutes to outline your synthesis essay, you can use the remaining 30 to 35 minutes to draft your essay and review it.

Since you'll outline your essay before you start drafting, writing the essay should be pretty straightforward. You'll already know how many paragraphs you're going to write, what the topic of each paragraph will be, and what quotations, paraphrases, or summaries you're going to include in each paragraph from the sources provided. You'll just have to fill in one of the most important parts of your synthesis—your commentary.

Commentaries are your explanation of why your evidence supports the argument you've outlined in your thesis. Your commentary is where you actually make your argument, which is why it's such a critical part of your synthesis essay.

When thinking about what to say in your commentary, remember one thing the AP Lang synthesis essay prompt specifies: don't just summarize the sources. Instead, as you provide commentary on the evidence you incorporate, you need to explain how that evidence supports or undermines your thesis statement . You should include commentary that offers a thoughtful or novel perspective on the evidence from your sources to develop your argument.

One very important thing to remember as you draft out your essay is to cite your sources. The AP Lang exam synthesis essay prompt indicates that you can use generic labels for the sources provided (e.g. "Source 1," "Source 2," "Source 3," etc.). The exam prompt will indicate which label corresponds with which source, so you'll need to make sure you pay attention and cite sources accurately. You can cite your sources in the sentence where you introduce a quote, summary, or paraphrase, or you can use a parenthetical citation. Citing your sources affects your score on the synthesis essay, so remembering to do this is important.

body-green-arrow-down

Keep reading for a real-life example of a great AP synthesis essay response!

Real-Life AP Synthesis Essay Example and Analysis

If you're still wondering how to write a synthesis essay, examples of real essays from past AP Lang exams can make things clearer. These real-life student AP synthesis essay responses can be great for helping you understand how to write a synthesis essay that will knock the graders' socks off .

While there are multiple essay examples online, we've chosen one to take a closer look at. We're going to give you a brief analysis of one of these example student synthesis essays from the 2019 AP Lang Exam below!

Example Synthesis Essay AP Lang Response

To get started, let's look at the official prompt for the 2019 synthesis essay:

In response to our society's increasing demand for energy, large-scale wind power has drawn attention from governments and consumers as a potential alternative to traditional materials that fuel our power grids, such as coal, oil, natural gas, water, or even newer sources such as nuclear or solar power. Yet the establishment of large-scale, commercial-grade wind farms is often the subject of controversy for a variety of reasons.

Carefully read the six sources, found on the AP English Language and Composition 2019 Exam (Question 1), including the introductory information for each source. Write an essay that synthesizes material from at least three of the sources and develops your position on the most important factors that an individual or agency should consider when deciding whether to establish a wind farm.

Source A (photo) Source B (Layton) Source C (Seltenrich) Source D (Brown) Source E (Rule) Source F (Molla)

In your response you should do the following:

  • Respond to the prompt with a thesis presents a defensible position.
  • Select and use evidence from at least 3 of the provided sources to support your line of reasoning. Indicate clearly the sources used through direct quotation, paraphrase, or summary. Sources may be cited as Source A, Source B, etc., or by using the description in parentheses.
  • Explain how the evidence supports your line of reasoning.
  • Use appropriate grammar and punctuation in communicating your argument.

Now that you know exactly what the prompt asked students to do on the 2019 AP Lang synthesis essay, here's an AP Lang synthesis essay example, written by a real student on the AP Lang exam in 2019:

[1] The situation has been known for years, and still very little is being done: alternative power is the only way to reliably power the changing world. The draw of power coming from industry and private life is overwhelming current sources of non-renewable power, and with dwindling supplies of fossil fuels, it is merely a matter of time before coal and gas fuel plants are no longer in operation. So one viable alternative is wind power. But as with all things, there are pros and cons. The main factors for power companies to consider when building wind farms are environmental boon, aesthetic, and economic factors.

[2] The environmental benefits of using wind power are well-known and proven. Wind power is, as qualified by Source B, undeniably clean and renewable. From their production requiring very little in the way of dangerous materials to their lack of fuel, besides that which occurs naturally, wind power is by far one of the least environmentally impactful sources of power available. In addition, wind power by way of gearbox and advanced blade materials, has the highest percentage of energy retention. According to Source F, wind power retains 1,164% of the energy put into the system – meaning that it increases the energy converted from fuel (wind) to electricity 10 times! No other method of electricity production is even half that efficient. The efficiency and clean nature of wind power are important to consider, especially because they contribute back to power companies economically.

[3] Economically, wind power is both a boon and a bone to electric companies and other users. For consumers, wind power is very cheap, leading to lower bills than from any other source. Consumers also get an indirect reimbursement by way of taxes (Source D). In one Texan town, McCamey, tax revenue increased 30% from a wind farm being erected in the town. This helps to finance improvements to the town. But, there is no doubt that wind power is also hurting the power companies. Although, as renewable power goes, wind is incredibly cheap, it is still significantly more expensive than fossil fuels. So, while it is helping to cut down on emissions, it costs electric companies more than traditional fossil fuel plants. While the general economic trend is positive, there are some setbacks which must be overcome before wind power can take over as truly more effective than fossil fuels.

[4] Aesthetics may be the greatest setback for power companies. Although there may be significant economic and environmental benefit to wind power, people will always fight to preserve pure, unspoiled land. Unfortunately, not much can be done to improve the visual aesthetics of the turbines. White paint is the most common choice because it "[is] associated with cleanliness." (Source E). But, this can make it stand out like a sore thumb, and make the gargantuan machines seem more out of place. The site can also not be altered because it affects generating capacity. Sound is almost worse of a concern because it interrupts personal productivity by interrupting people's sleep patterns. One thing for power companies to consider is working with turbine manufacturing to make the machines less aesthetically impactful, so as to garner greater public support.

[5] As with most things, wind power has no easy answer. It is the responsibility of the companies building them to weigh the benefits and the consequences. But, by balancing economics, efficiency, and aesthetics, power companies can create a solution which balances human impact with environmental preservation.

And that's an entire AP Lang synthesis essay example, written in response to a real AP Lang exam prompt! It's important to remember AP Lang exam synthesis essay prompts are always similarly structured and worded, and students often respond in around the same number of paragraphs as what you see in the example essay response above.

Next, let's analyze this example essay and talk about what it does effectively, where it could be improved upon, and what score past exam scorers awarded it.

To get started on an analysis of the sample synthesis essay, let's look at the scoring commentary provided by the College Board:

  • For development of thesis, the essay received 1 out of 1 possible points
  • For evidence and commentary, the essay received 4 out of 4 possible points
  • For sophistication of thought, the essay received 0 out of 1 possible points.

This means that the final score for this example essay was a 5 out of 6 possible points . Let's look more closely at the content of the example essay to figure out why it received this score breakdown.

Thesis Development

The thesis statement is one of the three main categories that is taken into consideration when you're awarded points on this portion of the exam. This sample essay received 1 out of 1 total points.

Now, here's why: the thesis statement clearly and concisely conveys a position on the topic presented in the prompt--alternative energy and wind power--and defines the most important factors that power companies should consider when deciding whether to establish a wind farm.

Evidence and Commentary

The second key category taken into consideration when synthesis exams are evaluated is incorporation of evidence and commentary. This sample received 4 out of 4 possible points for this portion of the synthesis essay. At bare minimum, this sample essay meets the requirement mentioned in the prompt that the writer incorporate evidence from at least three of the sources provided.

On top of that, the writer does a good job of connecting the incorporated evidence back to the claim made in the thesis statement through effective commentary. The commentary in this sample essay is effective because it goes beyond just summarizing what the provided sources say. Instead, it explains and analyzes the evidence presented in the selected sources and connects them back to supporting points the writer makes in each body paragraph.

Finally, the writer of the essay also received points for evidence and commentary because the writer developed and supported a consistent line of reasoning throughout the essay . This line of reasoning is summed up in the fourth paragraph in the following sentence: "One thing for power companies to consider is working with turbine manufacturing to make the machines less aesthetically impactful, so as to garner greater public support."

Because the writer did a good job consistently developing their argument and incorporating evidence, they received full marks in this category. So far, so good!

Sophistication of Thought

Now, we know that this essay received a score of 5 out of 6 total points, and the place where the writer lost a point was on the basis of sophistication of thought, for which the writer received 0 out of 1 points. That's because this sample essay makes several generalizations and vague claims where it could have instead made specific claims that support a more balanced argument.

For example, in the following sentence from the 5th paragraph of the sample essay, the writer misses the opportunity to state specific possibilities that power companies should consider for wind energy . Instead, the writer is ambiguous and non-committal, saying, "As with most things, wind power has no easy answer. It is the responsibility of the companies building them to weigh the benefits and consequences."

If the writer of this essay was interested in trying to get that 6th point on the synthesis essay response, they could consider making more specific claims. For instance, they could state the specific benefits and consequences power companies should consider when deciding whether to establish a wind farm. These could include things like environmental impacts, economic impacts, or even population density!

Despite losing one point in the last category, this example synthesis essay is a strong one. It's well-developed, thoughtfully written, and advances an argument on the exam topic using evidence and support throughout.

body-number-four-post-it-note

4 Tips for How to Write a Synthesis Essay

AP Lang is a timed exam, so you have to pick and choose what you want to focus on in the limited time you're given to write the synthesis essay. Keep reading to get our expert advice on what you should focus on during your exam.

Tip 1: Read the Prompt First

It may sound obvious, but when you're pressed for time, it's easy to get flustered. Just remember: when it comes time to write the synthesis essay, read the prompt first !

Why is it so important to read the prompt before you read the sources? Because when you're aware of what kind of question you're trying to answer, you'll be able to read the sources more strategically. The prompt will help give you a sense of what claims, points, facts, or opinions to be looking for as you read the sources.

Reading the sources without having read the prompt first is kind of like trying to drive while wearing a blindfold: you can probably do it, but it's likely not going to end well!

Tip 2: Make Notes While You Read

During the 15-minute reading period at the beginning of the synthesis essay, you'll be reading through the sources as quickly as you can. After all, you're probably anxious to start writing!

While it's definitely important to make good use of your time, it's also important to read closely enough that you understand your sources. Careful reading will allow you to identify parts of the sources that will help you support your thesis statement in your essay, too.

As you read the sources, consider marking helpful passages with a star or check mark in the margins of the exam so you know which parts of the text to quickly re-read as you form your synthesis essay. You might also consider summing up the key points or position of each source in a sentence or a few words when you finish reading each source during the reading period. Doing so will help you know where each source stands on the topic given and help you pick the three (or more!) that will bolster your synthesis argument.

Tip 3: Start With the Thesis Statement

If you don't start your synthesis essay with a strong thesis statement, it's going to be tough to write an effective synthesis essay. As soon as you finish reading and annotating the provided sources, the thing you want to do next is write a strong thesis statement.

According to the CollegeBoard grading guidelines for the AP Lang synthesis essay, a strong thesis statement will respond to the prompt— not restate or rephrase the prompt. A good thesis will take a clear, defensible position on the topic presented in the prompt and the sources.

In other words, to write a solid thesis statement to guide the rest of your synthesis essay, you need to think about your position on the topic at hand and then make a claim about the topic based on your position. This position will either be defending, challenging, or qualifying the claim made in the essay's prompt.

The defensible position that you establish in your thesis statement will guide your argument in the rest of the essay, so it's important to do this first. Once you have a strong thesis statement, you can begin outlining your essay.

Tip 4: Focus on Your Commentary

Writing thoughtful, original commentary that explains your argument and your sources is important. In fact, doing this well will earn you four points (out of a total of six)!

AP Lang provides six to seven sources for you on the exam, and you'll be expected to incorporate quotations, paraphrases, or summaries from at least three of those sources into your synthesis essay and interpret that evidence for the reader.

While incorporating evidence is very important, in order to get the extra point for "sophistication of thought" on the synthesis essay, it's important to spend more time thinking about your commentary on the evidence you choose to incorporate. The commentary is your chance to show original thinking, strong rhetorical skills, and clearly explain how the evidence you've included supports the stance you laid out in your thesis statement.

To earn the 6th possible point on the synthesis essay, make sure your commentary demonstrates a nuanced understanding of the source material, explains this nuanced understanding, and places the evidence incorporated from the sources in conversation with each other. To do this, make sure you're avoiding vague language. Be specific when you can, and always tie your commentary back to your thesis!

body-person-arrows-next

What's Next?

There's a lot more to the AP Language exam than just the synthesis essay. Be sure to check out our expert guide to the entire exam , then learn more about the tricky multiple choice section .

Is the AP Lang exam hard...or is it easy? See how it stacks up to other AP tests on our list of the hardest AP exams .

Did you know there are technically two English AP exams? You can learn more about the second English AP test, the AP Literature exam, in this article . And if you're confused about whether you should take the AP Lang or AP Lit test , we can help you make that decision, too.

Want to improve your SAT score by 160 points or your ACT score by 4 points? We've written a guide for each test about the top 5 strategies you must be using to have a shot at improving your score. Download it for free now:

Get eBook: 5 Tips for 160+ Points

Ashley Sufflé Robinson has a Ph.D. in 19th Century English Literature. As a content writer for PrepScholar, Ashley is passionate about giving college-bound students the in-depth information they need to get into the school of their dreams.

Student and Parent Forum

Our new student and parent forum, at ExpertHub.PrepScholar.com , allow you to interact with your peers and the PrepScholar staff. See how other students and parents are navigating high school, college, and the college admissions process. Ask questions; get answers.

Join the Conversation

Ask a Question Below

Have any questions about this article or other topics? Ask below and we'll reply!

Improve With Our Famous Guides

  • For All Students

The 5 Strategies You Must Be Using to Improve 160+ SAT Points

How to Get a Perfect 1600, by a Perfect Scorer

Series: How to Get 800 on Each SAT Section:

Score 800 on SAT Math

Score 800 on SAT Reading

Score 800 on SAT Writing

Series: How to Get to 600 on Each SAT Section:

Score 600 on SAT Math

Score 600 on SAT Reading

Score 600 on SAT Writing

Free Complete Official SAT Practice Tests

What SAT Target Score Should You Be Aiming For?

15 Strategies to Improve Your SAT Essay

The 5 Strategies You Must Be Using to Improve 4+ ACT Points

How to Get a Perfect 36 ACT, by a Perfect Scorer

Series: How to Get 36 on Each ACT Section:

36 on ACT English

36 on ACT Math

36 on ACT Reading

36 on ACT Science

Series: How to Get to 24 on Each ACT Section:

24 on ACT English

24 on ACT Math

24 on ACT Reading

24 on ACT Science

What ACT target score should you be aiming for?

ACT Vocabulary You Must Know

ACT Writing: 15 Tips to Raise Your Essay Score

How to Get Into Harvard and the Ivy League

How to Get a Perfect 4.0 GPA

How to Write an Amazing College Essay

What Exactly Are Colleges Looking For?

Is the ACT easier than the SAT? A Comprehensive Guide

Should you retake your SAT or ACT?

When should you take the SAT or ACT?

Stay Informed

ap lang synthesis essay time

Get the latest articles and test prep tips!

Looking for Graduate School Test Prep?

Check out our top-rated graduate blogs here:

GRE Online Prep Blog

GMAT Online Prep Blog

TOEFL Online Prep Blog

Holly R. "I am absolutely overjoyed and cannot thank you enough for helping me!”

What are your chances of acceptance?

Calculate for all schools, your chance of acceptance.

Duke University

Your chancing factors

Extracurriculars.

ap lang synthesis essay time

How to Write the AP Lang Synthesis Essay + Example

Do you know how to improve your profile for college applications.

See how your profile ranks among thousands of other students using CollegeVine. Calculate your chances at your dream schools and learn what areas you need to improve right now — it only takes 3 minutes and it's 100% free.

Show me what areas I need to improve

What’s Covered:

What is the ap lang synthesis essay, how will ap scores affect my college chances.

AP English Language and Composition, commonly known as AP Lang, is one of the most engaging and popular AP classes offered at most high schools, with over 535,000 students taking the class . AP Lang tests your ability to analyze written pieces, synthesize information, write rhetorical essays, and create cohesive and concrete arguments. However, the class is rather challenging as only 62% of students were able to score a three or higher on the exam. 

The AP Lang exam has two sections. The first consists of 45 multiple choice questions which need to be completed in an hour. This portion counts for around 45% of your total score. These questions ask students to analyze written pieces and answer questions related to each respective passage.  All possible answer choices can be found within the text, and no prior knowledge of literature is needed to understand the passages.

The second section contains three free-response questions to be finished in under two hours and 15 minutes. This section counts for 55% of your score and includes the synthesis essay, the rhetorical essay, and the argumentative essay.

  • The synthesis essay requires you to read 6-7 sources and create an argument using at least three sources.
  • The rhetorical analysis essay requires you to describe how a piece of writing evokes specific meanings and symbolism.
  • The argumentative essay requires you to pick a perspective of a debate and create an argument based on the evidence provided.

In this post, we will take a look at the AP Lang synthesis essay and discuss tips and tricks to master this part of the exam. We will also provide an example of a well-written essay for review.  

The AP Lang synthesis essay is the first of three essays included in the Free Response section of the AP Lang exam. The exam presents 6-7 sources that are organized around a specific topic, with two of those sources purely visual, including a single quantitative source (like a graph or pie chart). The remaining 4-5 sources are text-based, containing around 500 words each. It’s recommended that students spend an hour on this essay—15 minute reading period, 40 minutes writing, and 5 minutes of spare time to check over work.

Each synthesis essay has a topic that all the sources will relate to. A prompt will explaining the topic and provide some background, although the topics are usually broad so you will probably know something related to the issue. It will also present a claim that students will respond to in an essay format using information from at least three of the provided sources. You will need to take a stance, either agreeing or disagreeing with the position provided in the claim. 

According to the CollegeBoard, they are looking for essays that “combine different perspectives from sources to form a support of a coherent position.” This means that you must state your claim on the topic and highlight relationships between several sources that support your specific position on the topic. Additionally, you’ll need to cite clear evidence from your sources to prove your point.

The synthesis essay counts for six points on the AP Lang exam. Students can receive 0-1 points for writing a thesis statement, 0-4 based on the incorporation of evidence and commentary, and 0-1 points based on the sophistication of thought and demonstration of complex understanding.

While this essay seems extremely overwhelming, considering there are a total of three free-response essays to complete, with proper time management and practiced skills, this essay is manageable and straightforward. In order to enhance the time management aspect of the test to the best of your ability, it is essential to divide the essay up into five key steps.

Step 1: Analyze the Prompt

As soon as the clock starts, carefully read and analyze what the prompt asks from you. It might be helpful to markup the text to identify the most critical details. You should only spend around 2 minutes reading the prompt so you have enough time to read all the sources and figure out your argument. Don’t feel like you need to immediately pick your stance on the claim right after reading the prompt. You should read the sources before you commit to your argument.

Step 2: Read the Sources Carefully

Although you are only required to use 3 of the 6-7 sources provides, make sure you read ALL of the sources. This will allow you to better understand the topic and make the most educated decision of which sources to use in your essay. Since there are a lot of sources to get through, you will need to read quickly and carefully.

Annotating will be your best friend during the reading period. Highlight and mark important concepts or lines from each passage that would be helpful in your essay. Your argument will probably begin forming in your head as you go through the passages, so you will save yourself a lot of time later on if you take a few seconds to write down notes in the margins. After you’ve finished reading a source, reflect on whether the source defends, challenges, or qualifies your argument.

You will have around 13 minutes to read through all the sources, but it’s very possible you will finish earlier if you are a fast reader. Take the leftover time to start developing your thesis and organizing your thoughts into an outline so you have more time to write. 

Step 3: Write a Strong Thesis Statement 

In order to write a good thesis statement, all you have to do is decide your stance on the claim provided in the prompt and give an overview of your evidence. You essentially have three choices on how to frame your thesis statement: You can defend, challenge or qualify a claim that’s been provided by the prompt. 

  • If you are defending the claim, your job will be to prove that the claim is correct .
  • If you are challenging the claim, your job will be to prove that the claim is incorrect .
  • If you choose to qualify the claim, your job will be to agree to a part of the claim and disagree with another part of the claim. 

A strong thesis statement will clearly state your stance without summarizing the issue or regurgitating the claim. The CollegeBoard is looking for a thesis statement that “states a defensible position and establishes a line of reasoning on the issue provided in the prompt.”

Step 4: Create a Minimal Essay Outline

Developing an outline might seem like a waste of time when you are up against the clock, but believe us, taking 5-10 minutes to outline your essay will be much more useful in the long run than jumping right into the essay.

Your outline should include your thesis statement and three main pieces of evidence that will constitute each body paragraph. Under each piece of evidence should be 2-3 details from the sources that you will use to back up your claim and some commentary on how that evidence proves your thesis.

Step 5: Write your Essay

Use the remaining 30-35 minutes to write your essay. This should be relatively easy if you took the time to mark up the sources and have a detailed outline.  Remember to add special consideration and emphasis to the commentary sections of the supporting arguments outlined in your thesis. These sentences are critical to the overall flow of the essay and where you will be explaining how the evidence supports or undermines the claim in the prompt.

Also, when referencing your sources, write the in-text citations as follows: “Source 1,” “Source 2,” “Source 3,” etc. Make sure to pay attention to which source is which in order to not incorrectly cite your sources. In-text citations will impact your score on the essay and are an integral part of the process.

After you finish writing, read through your essay for any grammatical errors or mistakes before you move onto the next essay.

Here are six must-have tips and tricks to get a good score on the synthesis essay:

  • Cite at least four sources , even though the minimum requirement is three. Remember not to plagiarize and cite everything you use in your arguments.
  • Make sure to develop a solid and clear thesis . Develop a stable stance for the claim and stick with it throughout the entire paper.
  • Don’t summarize the sources. The summary of the sources does not count as an argument. 
  • You don’t necessarily have to agree with the sources in order to cite them. Using a source to support a counterargument is still a good use of a source.
  • Cite the sources that you understand entirely . If you don’t, it could come back to bite you in the end. 
  • Use small quotes , do not quote entire paragraphs. Make sure the quote does not disrupt the flow or grammar of the sentence you write. 

ap lang synthesis essay time

Discover your chances at hundreds of schools

Our free chancing engine takes into account your history, background, test scores, and extracurricular activities to show you your real chances of admission—and how to improve them.

Here is an example prompt and essay from 2019 that received 5 of the 6 total points available:

In response to our society’s increasing demand for energy, large-scale wind power has drawn attention from governments and consumers as a potential alternative to traditional materials that fuel our power grids, such as coal, oil, natural gas, water, or even newer sources such as nuclear or solar power. Yet the establishment of large-scale, commercial-grade wind farms is often the subject of controversy for a variety of reasons.

Carefully read the six sources, found on the AP English Language and Composition 2019 Exam (Question 1), including the introductory information for each source. Write an essay that synthesizes material from at least three of the sources and develops your position on the most important factors that an individual or agency should consider when deciding whether to establish a wind farm.

Source A (photo)

Source B (Layton)

Source C (Seltenrich)

Source D (Brown)

Source E (Rule)

Source F (Molla)

In your response you should do the following:

  • Respond to the prompt with a thesis presents a defensible position.
  • Select and use evidence from at least 3 of the provided sources to support your line of reasoning. Indicate clearly the sources used through direct quotation, paraphrase, or summary. Sources may be cited as Source A, Source B, etc., or by using the description in parentheses.
  • Explain how the evidence supports your line of reasoning.
  • Use appropriate grammar and punctuation in communicating your argument.

[1] The situation has been known for years, and still very little is being done: alternative power is the only way to reliably power the changing world. The draw of power coming from industry and private life is overwhelming current sources of non-renewable power, and with dwindling supplies of fossil fuels, it is merely a matter of time before coal and gas fuel plants are no longer in operation. So one viable alternative is wind power. But as with all things, there are pros and cons. The main factors for power companies to consider when building wind farms are environmental boon, aesthetic, and economic factors.

[2] The environmental benefits of using wind power are well-known and proven. Wind power is, as qualified by Source B, undeniably clean and renewable. From their production requiring very little in the way of dangerous materials to their lack of fuel, besides that which occurs naturally, wind power is by far one of the least environmentally impactful sources of power available. In addition, wind power by way of gearbox and advanced blade materials, has the highest percentage of energy retention. According to Source F, wind power retains 1,164% of the energy put into the system – meaning that it increases the energy converted from fuel (wind) to electricity 10 times! No other method of electricity production is even half that efficient. The efficiency and clean nature of wind power are important to consider, especially because they contribute back to power companies economically.

[3] Economically, wind power is both a boon and a bone to electric companies and other users. For consumers, wind power is very cheap, leading to lower bills than from any other source. Consumers also get an indirect reimbursement by way of taxes (Source D). In one Texan town, McCamey, tax revenue increased 30% from a wind farm being erected in the town. This helps to finance improvements to the town. But, there is no doubt that wind power is also hurting the power companies. Although, as renewable power goes, wind is incredibly cheap, it is still significantly more expensive than fossil fuels. So, while it is helping to cut down on emissions, it costs electric companies more than traditional fossil fuel plants. While the general economic trend is positive, there are some setbacks which must be overcome before wind power can take over as truly more effective than fossil fuels.

[4] Aesthetics may be the greatest setback for power companies. Although there may be significant economic and environmental benefit to wind power, people will always fight to preserve pure, unspoiled land. Unfortunately, not much can be done to improve the visual aesthetics of the turbines. White paint is the most common choice because it “[is] associated with cleanliness.” (Source E). But, this can make it stand out like a sore thumb, and make the gargantuan machines seem more out of place. The site can also not be altered because it affects generating capacity. Sound is almost worse of a concern because it interrupts personal productivity by interrupting people’s sleep patterns. One thing for power companies to consider is working with turbine manufacturing to make the machines less aesthetically impactful, so as to garner greater public support.

[5] As with most things, wind power has no easy answer. It is the responsibility of the companies building them to weigh the benefits and the consequences. But, by balancing economics, efficiency, and aesthetics, power companies can create a solution which balances human impact with environmental preservation.

More examples can be found here at College Board.

While AP Scores help to boost your weighted GPA, or give you the option to get college credit, AP Scores don’t have a strong effect on your admissions chances . However, colleges can still see your self-reported scores, so you might not want to automatically send scores to colleges if they are lower than a 3. That being said, admissions officers care far more about your grade in an AP class than your score on the exam.

Related CollegeVine Blog Posts

ap lang synthesis essay time

Nerdpapers logo

We have sent you an email with a 6 digit code to:

Didn't receive an email? Check your spam folder and mark the email as not spam!. If you Skip this step, you won't be able to receive order-related updates via email.

How to write an AP Lang synthesis essay: Format + Outline + Tips

ap lang synthesis essay time

The last component of your basic college English composition course is the AP Lang test. The most crucial—and sometimes challenging—component is the AP Language Synthesis Essay. It's the cornerstone of your future writing abilities, but if you're not prepared, it can be challenging. Additionally, each year's test details could vary. So, with the help of our coursework writers, let's explore what an AP lang synthesis essay is and how to write it. 

What is ap lang?

The AP Language exam is a bit lengthy. Students who want to pass this AP test need to be well-versed in multiple rubrics. Reading and writing are covered in the first half, while three distinct kinds of essays are covered in the second, which is a little more freeform. Of those three, the synthesis essay is the AP Lang rubric, which is the most interesting and, surprisingly, often the most challenging to tackle. We'll concentrate today on the AP Lang synthesis essay, especially to make sure you know what to expect on your test.

Understanding the AP Lang synthesis essay

Do you know what is a synthesis essay ap lang? Imagine someone is selling a mysterious remedy claiming it can cure any disease, but they won't reveal its ingredients or how it works. Would it be wise to buy this remedy? Probably not!

The same procedure applies when writing an essay. If you do not cite your sources for your thoughts, a reader won't believe even your ideas are the best ones. That's where the synthesis essay comes in! In a comprehensive essay, an argument is made on the basis of outside sources. Synthesis essays are used to develop compelling arguments for your viewpoints. You not only need to gather the information, but you also need to add your opinion. 

The goal of the AP synthesis essay rubric is to demonstrate your ability for in-depth source analysis and the creation of an original, well-reasoned thesis statement backed up by relevant evidence. Writing an essay of this kind requires both critical and creative thinking since you have to evaluate the data you have collected and combine it to create a fresh, original viewpoint.

If you find this assignment too complex, you can ask us, ' Write my essay for me ', and our writers will complete it for you.

Types of Synthesis Essays

Synthesis essays are of different types. But the most common are explanatory and argumentative synthesis essays. Let's have a look at them:

Explanatory synthesis essay

The purpose of an explanatory essay is to cover a subject, go into great depth about it, and then show a deep understanding of the subject. You can convince the reader about something without necessarily getting into a furious debate with them. All you have to do is compile an unbiased summary.

Argumentative synthesis essay

An argumentative essay contains personal opinions, but it should be as objective and fact based as possible despite the possibility of bias. You should stick to logical and objective facts when presenting your arguments rather than depending on your feelings. This strategy will strengthen your claim.

Key Components of an AP Lang Synthesis Essay Outline

When you go through the examples of synthesis essays, you will notice that their structure is almost similar to other essays. You have to do an in-depth analysis of the material you gathered before you start working on a synthesis essay ap lang outline. Keep in mind that if the document is properly structured, then it must receive good grades. If you don't format your essay correctly, you won't get good marks even if you write a superb one. You must ensure that you have a thorough understanding of how to write a synthesis essay ap lang.

A lot of time is required to write a synthesis essay outline for the AP Lang test. Simply follow the below steps:

Introduction:

Write about the topic you're going to cover. Add a strong thesis statement. Your thesis statement should cover three ideas on the selected topic. Moreover, it should also include some background details and a strong attention-grabbing hook statement.

The body paragraphs (3 to 4 paragraphs):

As per the standard essay outline, you must write three paragraphs in the body section of your synthesis essay. Start each paragraph with a new thought or idea. If you use someone else's words exactly, put them in quotes. Also, the evidence must be from a reliable source. Any information that is not your own should be cited, and your sources should all be listed in your works cited page or bibliography.

Conclusion (1 paragraph):

This section is dedicated to wrapping up your paper and providing a logical conclusion. Only tie up the topics you covered in the introduction and body, without introducing any new ideas. Your conclusion statement must be strong enough to leave a lasting impression on the reader.

How do you write a synthesis essay introduction?

The most challenging aspect of writing is often getting started. When your mind is teeming with ideas, it can be overwhelming to decide where to begin. Students often spend numerous hours brainstorming ideas. An effective approach is to address this question: 'What would grab my attention if I were the reader"? When you're coming up with ideas for how to compose the paper, keep in mind the specific requirements of the synthesis essay. You can begin in a few different ways. First method is to present a fascinating fact that not everyone is aware of. The second is to start the essay with a well-known quotation related to the subject. The success of your work will mostly depend on the first impression you make on the reader. So, try to grab their interest quickly.

You can also learn “ how to write a research paper introduction ” from our latest guide.

Step-By-Step Guide on How to write a good synthesis essay AP lang

Here are some good techniques for writing a synthesis essays:

Thoroughly read assignment details

Think about the requirements for the assignment. It is very important to understand the assignment completely. Teachers will frequently ask a question regarding a particular subject. They may then assign you a number of readings from articles, academic journals, or other sources to help you gain information. Knowing what the assignment entails will help you make sure you focus on the appropriate material in your readings.

Choose or read about the topic

Read the materials carefully, regardless of whether your instructor assigns you to read a certain amount or requires you to conduct your own research. To fully understand the authors' points of view and their connections, think about annotating each text. One of the most important aspects of synthesis essays is the ability to contrast and compare ideas from multiple sources.

Make a strong thesis statement

Take time to read each source deeply and create a synthesis essay AP lang thesis statement. Do not overlook minute details. The thesis statement is the baseline of your writing. Write it clearly and concisely. Do not overthink too much because you're not adding any details at this point.

Create synthesis essay outline ap lang

Making an outline will assist you in organizing the structure of your essay. You can outline your topic and any supporting details in a structured format with Roman numerals. There may be subcategories inside each where you can put references to the different articles and make particular references to points you will make. By citing these sources, you can make sure that your essay includes passages or details from each article.

Compose synthesis essay introduction

A paper's introduction is its opening paragraph. Its primary goal is to introduce the paper's basic idea, include any background information that is required, and, ideally, grab the reader's interest. You should have a compelling thesis statement in the first paragraph. This is where you will introduce your argument or the point of view you are investigating.

Add body paragraphs to your essay

The three primary components of a strong body paragraph are a topic sentence (also known as a key sentence), relevant supporting phrases, and a closing (also known as a transitional) sentence. This format provides clear, succinct information while maintaining your paragraph's focus on the main point. The various components and justifications of your thesis, together with supporting data from each source, should be covered in detail in each body paragraph. Describe the theme that runs through your materials and how your text relates to them. Add counter arguments as well as how your source material may dispute those claims while supporting your own.

Write conclusion

The last section of a research paper, essay, or article that provides a summary of the entire work is called the conclusion. Your synthesis essay's conclusion paragraph should repeat your thesis, provide a summary of the main points you raised throughout the essay, and provide your assessment of the main idea.

Make a bibliography

An essay's bibliography is a list of the sources you consulted while writing it. You may compose this on a formal work cited or references page at the conclusion of the essay, depending on the essay type. Try including any works that you have quoted or paraphrased in order to provide context.

If you don't know how to cite sources in AP Lang synthesis essay, then avail of our annotated bibliography writing service to cite your sources correctly. 

When writing is complete, go over the assignment once more to see if there are any formatting requirements. Depending on your field, you may frequently submit essays using common formatting styles like MLA or APA. This may change depending on the kind of essay you write because synthesis essays can be written on a wide range of topics.

Recommended Article: “ MLA essay format ” 

Before submitting or presenting your assignment, make sure to proofread it several times. Sometimes, the entire core of an argument or opinion can be altered or discredited by a few misplaced words or grammatical faults. Make sure your syntax, grammar, and writing style are as precise and understandable as you can. This will make you seem like a reliable source.

Synthesis Essay AP lang rubric

Generally, a synthesis essay rubric assesses multiple factors, such as your ability to compile and integrate several sources, your ability to build a solid argument, and your proficiency with language and mechanics. Although rubrics differ from instructor to instructor, a simple AP synthesis essay rubric includes the following:

  • Your argument's cohesion and clarity
  • Ability to back up your claims with proof
  • Language and mechanics used
  • Combining ideas from several sources
  • Information synthesis from several sources

To make sure you're fulfilling all the requirements for the assignment, make sure you go over any rubrics your professor or teacher may have provided. As an alternative, you can hire our team of qualified writers to complete your write my term paper  request without sacrificing any of your own work.

Tips for Synthesis Essay AP Lang

Now that you know how to write a synthesis essay in AP lang properly, your chances of receiving an A are significantly increased. Use these synthesis essay tips in AP Lang to ensure that your project will receive a high score. You can become an expert writer by adhering to them.

Pick the right subject

As a student, you may be motivated to write on a topic about which you are unfamiliar in order to widen your horizons. Learning new things is perfectly acceptable, but it should happen on your own time rather than while you are completing a paper for a grade. Write an essay based on your knowledge; we assure you that you will learn a great deal of new information when you research the topic. Focusing on improving your writing abilities and achieving a good mark should be your main objectives; writing on an entirely different subject makes it more likely that you won't achieve an A.   

Refer credible sources

One of the main issues facing academia today is that new students frequently quote Facebook, blogs, and other unreliable media sources in their papers because they are unsure of what makes an authentic source. When writing a paper, refer only to books, academic journals, the internet, and other similar resources.

Properly cite your sources

Making improper or missing citations and bibliographies will always result in a failing assignment. Failure to adhere to the professor's directions is another factor that frequently gets students into trouble. This frequently occurs when a student chooses to cite in APA or Harvard styles, but the professor prefers another format, like MLA. Never carry out this. Regardless of whether you agree with your professor's instructions or not, you should always obey them.

Read your essay aloud

Speaking out loud while reading an essay can help you identify sentences or paragraphs that need clarification or correction. Try doing this to find any last-minute structural or grammar errors.

Synthesis Essay Do's and Don'ts

  • Leave enough time for proofreading and editing your essay.
  • Make a proper schedule for how much time you are required to write each section.
  • Try to use MLA or Chicago style to format your essay.
  • Add as many arguments as you can to support your claim, but all must be solid and relevant to the topic.
  • Read the source material thoroughly.
  • Private anecdotes should not be used to support your argument unless the essay genre expressly permits it. These don't provide a strong enough case.
  • Don't get too deep into the essay topic; instead, immerse yourself and show your thorough understanding of the subject.

AP Lang Synthesis Essay Prompts

Here is a list of AP synthesis essay prompts:

  • Analyze the impact of technology on modern society.  Use evidence from at least four sources to support your viewpoint.
  • Discuss the government role in climate change.
  • Explore the pros and cons of social media in the context of interpersonal communication.
  • Analyze the relationship between education and income inequality.
  • Consider the effects of globalization on the economy. Synthesize information from at least three sources to make your case.
  • Discuss the effects of climate change on global agriculture. Cite evidence from at least four sources to support your argument.
  • Consider the effects of widespread monitoring programs on people's civil liberties and right to privacy while balancing potential benefits for national security against worries about privacy invasion.
  • Analyze the impact of immigration on cultural diversity in the United States.
  • Discuss the ways that media sources, such as news sites, social media, and advertisements, affect societal attitudes, beliefs, and actions as well as public opinion.

Final Thoughts

Now we've come to the end of the blog; you should not be feeling stressed by the thought of writing an AP Lang synthesis essay. Moreover, you already know that analyzing sources, creating a strong outline, and skillfully integrating them into your writing are essential to success.

So don't be afraid, brave writer! You're ready to start writing a successful paper now because you have these synthesis essay writing strategies at your hand. Alternatively, you may simply buy an essay to wow your teachers!

Table of Contents

Persuasive essay topics – how to choose one for you, how to write a persuasive essay- expert tips.

ap lang synthesis essay time

Tackling the AP English Language and Composition essays: part 1

Statistical Mediation & Moderation in Psychological Research-Apr-06-2021-08-55-30-55-PM

So, what are the three AP Lang Essays? The College Board shares a lot of general information about these essays on its website, as well as a large number of excellent sample essays. I suggest you take the time to review all of that material, here. But here’s my primer:

On the AP Lang Exam, there are three essays to write, all in a row (during the second half of the exam, after an initial multiple-choice portion). They are:

  • The Synthesis Essay: You’ll be given a general topic or question for debate (like: should public libraries continue to exist? Or: is eminent domain just?). Multiple short sources taking positions on that topic will follow the prompt. You will then be asked to write your own, short essay taking a position on the topic, citing at least three of the sources that you read.
  • The Rhetoric Essay: You’ll be given a short, rhetorically interesting passage, either taking a position on a topic, telling a story, or performing some other function. You will then be asked to write a short essay analyzing this passage’s use of language/rhetorical approach.
  • The Argument Essay: You will be given some position, usually stated in some brief excerpt from an author’s work. For example, you might be given an excerpt from Proust that suggests that people often regret their choices, or an excerpt from Eleanor Roosevelt praising the virtue of courage. You will then be asked to take your own position on the topic. This time, you won’t be given sources to help you make your arguments; all of your arguments must come from your own brain. 

The scoring rubric for each essay is roughly similar, with six possible points awarded: there is one point for argument, four points for evidence and analysis, and one point for “sophistication.” What this means is that, in brief, you need to do three things on every essay to get a perfect score: 

  • Have an argument.
  • Back up your argument with evidence and analyze how that evidence supports your argument.
  • Have an ineffable, excellent quality to your writing, a sort of dexterity of mind and language, for which the scorers have reserved one, sacred point. 

You can’t really control whether or not you can achieve #3, and a lot of that will be based on your prior level of experience writing/reading; but you can control whether or not you achieve #1-2. So, a high score is totally within your power! The TLDR version of this post is: make a clear argument and back it up with concrete, analyzed evidence. But, of course, that’s not as easy as it looks, and I have many more thoughts on how to actually achieve it, and achieve it well...

The six major components of successfully writing a timed essay on an exam are:

  • Organizing your time
  • Reading and Annotating
  • Outlining Part 1: Thesis
  • Outlining Part 2: Structure
  • Writing Part 1: Paragraphs (Intro, Evidence, Analysis, Conclusion)
  • Writing Part 2: Sentence by Sentence

#1 Organizing your time

On the AP Lang exam, you get a total of 2 hours and 15 minutes to write your three essays. This time is split into chunks. First, there is a 15 minute “reading period”; next, there is a 2 hour “writing period.” What this seems to imply is that the exam would like you to read all of the questions and their supplemental texts (the Synthesis Essay question and texts, the Rhetoric Essay question and passage, the Argument Essay question and short question blurb) in the 15 minute reading period, and then proceed to write the essays, in response, in the two hour writing period. This, however, is obviously an insane approach. For one thing, it’s kind of impossible: no one could keep the details of three different essay questions and associated readings together in their head all at once. For another, it’s really time inefficient: if you read all the material for all three essays first, you’re going to have to go back to it, a lot, each time you start to write a new essay, to jog your memory. Basically, no one in their right mind would (or does) advise this approach. And even the College Board seems to know it makes no sense, because they allow you to continue reading and referring to the questions and texts after the reading period. 

What you should do instead? Simply treat the whole 2 hours and 15 minutes as a single time block. Divide it into three units of 45 minutes. Then, read and answer each of the three questions one after the other, giving 45 minutes to each. Start with the Synthesis Essay, followed by the Rhetoric Essay, and then the Argument Essay. 

Your process should look like this: during the 15 minute reading period, begin work on the Synthesis Essay by reading the question and texts and planning that essay. Then, when the 2-hour timer starts, devote the first 30 minutes to actually writing that essay. Next, spend 45 minutes reading the Rhetoric Essay question and passage, and writing the Rhetoric Essay. Finally, spend the last 45 minutes reading the Argument Essay question/blurb and then writing the Argument Essay. The Argument Essay should actually take you less time than the first two, which means you should end up with 5-10 minutes to proofread your other essays. That said, I advise that you leave time at the end of each 45-minute block to check over each individual essay. 

Now let’s talk about the Rhetoric Essay in particular. How should you organize your 45 minutes here? I suggest mapping out your time roughly like this: take about ten minutes to read the passage, take notes, and brainstorm; then, take about five minutes to make an outline for your essay; next, take about twenty to twenty-five minutes to write. Leave an extra five to seven minutes at the end to re-read and edit your work. As you practice, you might notice that slightly different divisions of time work best for you – feel free to be flexible! You don’t have to stick to your timetable exactly . BUT you should try to stick to a version of this timetable so that you have enough time for each of the steps. How? Watch the clock!

#2 Reading and Annotating

The Rhetoric Essay asks you to analyze the language or rhetoric that a passage uses to achieve its ends. In your first ten minutes of reading, you should be keeping an eye out for two things: 

  • What is this passage trying to achieve? Is it trying to persuade the reader of an argument (often the case)? Is it trying to entertain the reader with a story (sometimes the case)? Is it trying to make the reader laugh? Is it trying to make the reader think? Identify the passage’s main purpose.
  • What rhetorical methods or devices does the passage use to achieve its aims? What exactly is it doing to achieve its aims? Yes, you should be watching out for rhetorical devices that already have fancy names, like “allusion” or “alliteration,” but you should also be using your OWN language/descriptive powers to identify the passage’s methods. You might, for example, note things like: “makes argument largely through anecdote” or “addresses counterarguments” or “lists so many absurd situations that they start to feel normal.” Try to identify not just rhetorical methods the passage uses, but also the central ones it uses.

To achieve this, I suggest proceeding as follows: read one paragraph. Once you’re done, stop, reflect, and note (in the margins) the most important rhetorical devices the passage used to achieve its aims (as far as you understand them thus far). Do this for each paragraph you read. Once you’re done, you should have a handy list in the margin of rhetorical tactics the passage uses. Which ones, looking back, seem to come up the most frequently? Which ones, even if they don’t come up frequently, seem particularly central to the passage’s aims? The tactics you identify will soon play a role in your essay’s thesis. 

Next, you’ll be ready to write an outline for your essay, mapping out (as best you can) its thesis and structure. In the next blog post , we’ll begin with that step. 

Related Content

ap lang synthesis essay time

How to Write the AP Lang Synthesis Essay

ap lang synthesis essay time

AP Lang test is the logical conclusion to the introductory college English composition course. And its most important (and often difficult) part is the AP Lang synthesis essay. Despite it being the very basic layer of your future composition skills, it’s a very complicated challenge to approach unprepared. Besides, it's details may change year to year. So let’s have a look with our coursework writing team at what your AP Lang exam 2022 might look like.

What is AP Lang?

AP Lang is a relatively lengthy test. There are several AP rubrics that a student must be well-versed in to hope to pass it. The first section includes reading and writing, while the second is slightly more freeform and includes three different types of essays.

Among those three, the most interesting and, coincidentally, oftentimes the hardest to deal with is the AP Lang synthesis essay rubric. Today will focus on it specifically to make sure you know exactly what you’re going to be facing during your test.

What Is a Synthesis Essay AP Lang?

At its core, the AP Lang synthesis essay is a pretty straightforward part of the AP Lang test. It might look pretty similar to the reading section of the exam. However, simply finding the right information isn’t enough. When writing a synthesis essay, you should not only gather the data but also distill it into your personal opinion.

This fine line may seem difficult to spot, but it is there. And it’s that small difference that can make or break your exam run. So try to follow the steps one by one and not lose focus. Writing a good synthesis essay is as easy as following the rules. If you feel this task is too difficult for you, you can leave us your ' write an essay for me ' request and we will do it for you.

AP Lang Synthesis Essay Outline

Looking through AP Lang essay examples, you might notice that the overall structure doesn’t really differ too much from your standard essay outline. You have your introduction, your body, and your conclusion. But the important thing to note is where your arguments are supposed to come from.

You’re not supposed to just go off on a rant. The task requires you to base your supporting evidence on at least three sources. And you will have to ensure your essay has solid roots. Here’s what a basic AP Lang exam synthesis essay outline should look like:

  • Introduction

Provide sufficient context for the topic you are about to cover. You can do a quick overview of prevailing opinions you have grasped while browsing through your source materials.

Write a short and compelling thesis statement. This will be your ground zero for the rest of the essay. So make sure it reflects your opinion. What is a thesis statement you can read in our special article.

  • Body Paragraphs

Dedicate at least one paragraph to every source you’re using. Start with presenting the evidence you have gathered from that source and go on to explain how it formed your opinion on the topic and why it should be considered.

Quickly go through your line of reasoning and reinforce what you have already covered. Finish up with restating your thesis as you’re supposed to logically arrive at it after all the evidence you have presented. That’s how you write a conclusion properly.

Different Forms and Types of Synthesis Essay: Explanatory vs. Argumentative Synthesis Essays

When it comes to writing a synthesis essay AP Lang, there are several types of essays you should consider. The most common ones are the AP Lang argument essay and explanatory essay. The clues as to how each of them should look are hidden within their names but let’s go over them to clear any confusion.

An explanatory essay’s goal is to go over a certain topic, discuss it in detail, and ultimately show a high level of understanding of the said topic. You don’t necessarily have to get into a heated argument with the reader trying to convince them of something. All you need to do is create an impartial overview.

On the other hand, an argumentative essay has to do with personal opinions. And while there is a time and a place for bias, it still has to be as impartial and factual as possible. When proving your point, try not to devolve into emotional arguments but stick to logic and cold truths. This will make your argument way more solid.

Synthesis Essay Structure

In the general case, you don’t really need to look for a synthesis essay AP Lang example to get a solid grasp on how its structure should look like. You can safely fall back on your high school essay writing knowledge, and you’ll be mostly safe.

What you should pay attention to is your writing style and content. A synthesis essay is identified less by its structure and more by the way you form and present your arguments to the reader. It’s when you get a specific essay type (like an argumentative essay) that you should pay attention to slight changes in format.

Argument Essay Structure

The best way to understand argumentative essay structure is to study any well-written AP Lang argument essay example. Standard AP Lang essays have very distinctive features that are very easy to spot and emulate. They follow a very rigid form and employ specific rhetorical devices that you’ll be able to pick up after you analyze them once or twice quickly.

How Many Paragraphs Should an AP Lang Synthesis Essay Be?

The number of paragraphs in an AP Lang synthesis essay can indeed make a difference. Your arguments should be concise and pointed. Spreading them out throughout many paragraphs may seem like a good idea to fill in the space. But it’s actually detrimental to your final score. You can get a basic understanding of what your score is going to be using an AP Lang score calculator.

The same goes for too few paragraphs. Don’t even try to squeeze your entire line of thought into a single body paragraph. Generally, the minimum number of sources you should address is three. Any less, and you are getting a lower score. So try to keep it somewhere in the middle. Three to five body paragraphs is an optimal number. Don’t forget to add an intro and a conclusion to it and you’re all set. A well-written essay has a clear and easily identifiable structure.

How to Write AP Lang Synthesis Essay: Guide

How to Write the AP Lang Synthesis Essay

In order to write a decent essay, all you have to do is follow these simple steps. Performing a rhetorical analysis essay example, AP Lang won’t give you insight into how it was built from the ground up. But looking at this list might.

Step 1. Read the Prompt

It may sound like a no-brainer. But it’s actually more important than you can imagine. Don’t skip right past this step. It’s very easy to misunderstand the task under stress. And if you do slip up in the beginning - the entirety of your work after that is wasted.

Step 2. Analyze the Sources Carefully

The same goes for your sources. Take your time reading them. Try to spot every smallest detail, as even a single one can help you better incorporate your evidence into the body of your essay. You can begin outlining the general points of your essay in your head at this point.

Step 3. Come Up with a Strong Thesis Statement

Your thesis statement is the baseline of your writing. Make it short and clear. Try not to overthink it too much.

Step 4. Fill in Your Essay Outline

Start filling out your outline step by step. You don’t have to go from top to bottom. If you feel like you’re struggling - skip to the next part and return to the problem paragraph later. The use of rhetorical devices AP Lang is also pretty important. So once you flesh out your essay a bit, spend some time trying to come up with the perfect wording.

Step 5. Finalize

The first finished version of your essay is a draft. Don’t be hasty to turn it in. Read over it a couple of times. Make sure everything is in order. You can switch some of the parts around or rewrite some sections if you have the time. Ideally, at this stage you should have enough time to eliminate all grammatical errors that may still be present in your essay. Polish it to perfection.

Useful Tips

Here are some useful tips that might make the writing process a bit easier for you:

  • Use either APA or Chicago style to cite your sources
  • Have a schedule to understand how much time you have for each section
  • Leave as much time as you can for editing and proofreading
  • You can never over study the source material. Spend as much time as you can reading into it
  • Don’t linger on the surface of your essay subject. Dive in and show your complex understanding of the material
  • Avoid using private life anecdotes to support your case unless the essay type specifically allows it. These don’t make for a convincing argument.
  • Use as many supporting arguments as you can but make sure they are actually solid and relevant to your thesis
  • Check with your thesis from time to time. The entirety of your text should align with it

Need help with academic deadlines?

Falling back on your deadlines? Use our term paper writing services to relieve you while you get back on your feet.

AP Lang Essay Prompts

Here are some interesting prompts. Some of them could be found in the previous iterations of the test; you may have spotted them in some of the AP Lang essay examples. Others are there to help you practice for the AP Lang exam 2022.

  • The John F. Kennedy Presidential Library and Museum, dedicated in 1979, was founded in memory of the president and contained archives pertaining to his administration. On June 24, 1985, then President Ronald Reagan joined members of the Kennedy family at a fundraising event to help the Kennedy Library Foundation create an endowment to fund and support the presidential library. The following is an excerpt from the speech Reagan gave at that event. Read the passage carefully. Write an essay that analyzes the rhetorical choices Reagan makes to achieve his purpose of paying tribute to John F. Kennedy.
  • On August 29, 2009, then-President Barack Obama delivered a eulogy at the funeral of Senator Ted Kennedy in Boston, Massachusetts. Kennedy served in the United States Senate from 1962 until his death. Obama served with him in the Senate from 2005 until Obama was elected president in 2008. The following is an excerpt from Obama’s speech. Read the passage carefully. Write an essay that analyzes the rhetorical choices Obama makes to achieve his purpose of praising and memorializing Kennedy.
  • On April 9, 1964, Claudia “Lady Bird” Johnson, who was at the time the First Lady of the United States, gave the following speech at the first-anniversary luncheon of the Eleanor Roosevelt Memorial Foundation. The foundation is a nonprofit division of the Franklin D. Roosevelt Presidential Library dedicated to the works of former First Lady Eleanor Roosevelt, who passed away in 1962. Read the passage carefully. Write an essay that analyzes the rhetorical choices Johnson makes to achieve her purpose of paying tribute to Eleanor Roosevelt.

In your response, you should do the following:

• Respond to the prompt with a thesis that analyzes the writer’s rhetorical choices.

• Select and use evidence to support your line of reasoning.

• Explain how the evidence supports your line of reasoning.

• Demonstrate an understanding of the rhetorical situation.

• Use appropriate grammar and punctuation in communicating your argument.

AP Lang Essay Example

Here is a decent if a bit shortened, AP Lang rhetorical analysis essay example you can use for reference.

Literature to Prepare for AP Lang

How to Write the AP Lang Synthesis Essay

And here is a list of some great AP Lang books that will help you prepare for the exam. Not all of them are immediately useful, but most will help you enhance your writing and analytical abilities to get a better score in the end.

  • The Odyssey
  • Don Quixote
  • A Midsummer Night's Dream
  • Pride and Prejudice
  • Wuthering Heights
  • Oliver Twist
  • Crime and Punishment
  • Adventures of Huckleberry Finn

If you have thoughts of "who could do my paper for me," do not forget that you can contact us. Or, if you have a finished paper and you need to make edits to it, leave us a ' rewrite my essay ' request and we will do it as soon as possible.

Related Articles

How to Write a Deductive Essay with Expert Insights

Fiveable

Find what you need to study

Practice Quizzes

Tell your fav AP teacher to apply.

(They get hired, you get $100!)

Synthesis Overview

10 min read • november 18, 2021

Justin Nazario

Justin Nazario

Overview of the Synthesis Question

Section II of the AP English Language and Composition exam includes three free-response questions that you must answer in 2 hours and 15 minutes.

This guide will focus on Question 1 of Section II of the exam, the Synthesis question . As with all AP exams with free-response questions, the Synthesis question has its own rubric and scoring that we will detail later in this guide. 

To summarize, however, your essay should include/ demonstrate the following:

An easy to identify thesis 

Use of three or more of the provided sources

Explain how the sources used defend the claim in a complex manner

Writing that is sophisticated and collegiate

In the sections that follow, we will go over exactly what each part means. One thing to keep in mind is that the sources you choose should only strengthen your claim-- not step in and be the claim. Avoid overly citing from the sources to the point that your voice takes the backseat.

Luckily, the same skills of sophistication and complexity translate into the other essays you’ll write for this exam. Once you have developed your own voice, the rest is a matter of organization.

As stated before, you have 2 hours and 15 minutes to answer all three of your free-response questions. It seems like a lot, but it flies. To prevent getting behind schedule, it’s important to manage your time wisely.

A good breakdown to consider when pacing yourself is the following:

10 min. (to read sources) + 5 min. (planning) + 35 min. (writing) = 50 min.

How to Rock the Synthesis Question: The Rubric

The synthesis question is scored on a six-point rubric , and each point can be earned individually. This means that you can get points in one category, but not in others. It all depends on how well you accomplish each level on the rubric .

The Synthesis Question Rubric

Your thesis is the statement of your essay that introduces your claim to the reader. This is where you come forward and explicitly say: here is my position on the argument, and here are my reasons for feeling this way. 💭Above all else, you must respond to the prompt in its entirety. 

As in most essays, the introduction is recommended to be in the opening paragraph of your essay. ☝If it’s not in the introduction, you run the risk of confusing your reader, but your thesis can be anywhere in your essay. It can be as long as you’d like, so long as you present your main ideas in the order you will be discussing them in.

In order to receive the point, you need to both answer the prompt and present your own argument and claim to said prompt. A simple way to do so is to use words from the prompt to drive your thesis forward, but avoid just restating the thesis without adding your claim . You’ll lose out on the point if you forget to weave your argument into the thesis.

Your thesis and introductory paragraph are really where you introduce your style and voice as a writer. You have the opportunity to speak to your reader-- say something. Answer the prompt in complex, rich sentences that convey your use the sources to their highest potential. 👏

A great thesis does not have to be a paragraph long: as long as it answers the prompt, you’ll be alright!

Evidence and Commentary

This section on the rubric is split up into two categories: use of sources and commentary on the sources.

The College Board requires that you use at least three of the sources in order to earn the maximum amount of points. To “use” a source, you must cite text from the source or paraphrase an idea expressed by the author of the source, and then must explain its significance to the overall claim. (More on that in a moment.)

You must also establish a line of reasoning that the sources answer and/or incorporate into your elaboration. To make it a bit simpler, you need to explain how the source proves or challenges your claim. This can be accomplished in one sentence or several-- regardless, you need to explain why you chose to use that source to prove that claim. 

The second part of this category is the commentary section. Here, you must consistently establish the line of reasoning for each of the sources you introduce and do so with complexity. In all reality, this is just making sure that you are using each source for a reason, and not just fact-dropping information to earn the point. 

https://firebasestorage.googleapis.com/v0/b/fiveable-92889.appspot.com/o/images%2Fdownload-24.png?alt=media&token=22532fea-589c-4e86-aac9-8831dfe52dbb

An easy way to do this is by prefacing your citation with how the source relates to your argument, and then elaborating afterward.  Consider this example:

“The indoctrination of immigrants into American society is representative of a divide in American politics and culture, a line created by the two party system. (Source 2) Through the conditioning of immigrants to the ways of American society, there is a systematic erasing of native culture and ways in order to push American agendas onto people of other backgrounds and identities...”

The example drops the citation right in the middle of the paragraph in order to introduce the paraphrased idea, but divide it from the elaboration that follows:

Sophistication

The final row in the rubric is sophistication , or the level and complexity of your writing. This point is earned over the course of your essay and must be consistent in order for you to get the point.

This one is a little more complex to earn than some of the other points on the rubric . Contrary to the other rows, this is not something you need to directly set out to do, but something that needs to be developed over the course of your essay-- when you read a well-crafted sentence, you can tell. When you don’t read a well-crafted sentence, you can tell.

College Board has 4 notes on responses that typically earn this point:

Typically notice variations and conflicts within the sources , and explore said variations and conflicts

Express the restrictions of a source’s argument and does so within a larger scope and context

Demonstrate specific and powerful use of language so as to express professionalism and maturity

Use voice that is consistently lively yet coherent

Let’s break down each bullet.

The first bullet states is asking that your response acknowledges the difference between sources. Let’s say Source A is about how peanut butter is good for dogs but Source B says that peanut butter is actually harmful for dogs-- by expressing the counterpoints of the two sources, and discussing the broader context of the source and arguments presented in the two, you are demonstrating sophistication and can earn the point. The ‘explore’ part of the bullet is what makes or breaks it.

Make sure you don’t just drop things without explaining their significance or value!

The second bullet is relating the sources and information presented in them to both one another and the overall prompt. Ask yourself: What does this source talk about that this one doesn’t? How is the scope of this source relating to the prompt? What does this source say that this one builds off of? It’s about finding relationships between the sources and how, together, they make a set and rely on one another for validation or dejection. 👪

The third and fourth bullets are notes on your writing. The College Board wants to read essays and responses that are high quality and complex, not ones that lack development or are lackluster. They are really looking for responses that feel whole and complete, expressing entire thoughts rather than fragments of ideas that can get scattered and lost in translation. 

This mainly comes with practice and reading your peers’ work. Look for things such as sentence structure, diction, and punctuation. Do most of their sentences follow the same order and flow? Do they use the same three words to describe one thing or are they using a wide array of vocabulary? Think of how you can apply these things to your own writing, as well.

How to Rock the Synthesis Question - Process

Before you start writing....

Take time to plan your essays. If you just jump into writing without jotting down some ideas or a battle plan, you’re going to find yourself lost in the middle of your body paragraphs .

https://firebasestorage.googleapis.com/v0/b/fiveable-92889.appspot.com/o/images%2Fdownload-25.png?alt=media&token=cc8dafca-19cf-410e-814c-75084b98d8f1

A very simple idea for planning your essay is by using a template:

Main Idea #1

Supporting Detail #1

Evidence #1

Evidence #2

Elaboration (2-3 Sentences)

Supporting Detail #2

By organizing your ideas into an umbrella shape, you can get an idea of how your essay is going to read by the progression of your ideas. Remember that the order you present your ideas in must be the order you discuss them!

Another tip is to be 100% of what it is the prompt is asking of you. If the prompt is asking you to develop an argument or position on an event or idea, do exactly that. The sources tend to lend themselves towards one side of the argument, so be sure that whatever side you pick is well-supported with evidence from the sources. You can’t use any outside knowledge or anything that is not directly stated or implied by the sources. 

As mentioned before, it is extremely useful to use words in the prompt to formulate your thesis.

For example, if the prompt asks you what a country needs to consider before it engages in war with another country, you could formulate your thesis by saying “prior to engaging in war with another country, one must consider…” in order to directly respond to the question. This avoids confusion and allows you to easily pinpoint, for yourself, your thesis.

Think of all of Section 2 as a speech– this is the only section of the exam where you get to speak to the scorers. They are reading your handwriting, seeing your words and erase marks: make an impression! They are scored by a rubric , but they are also looking for voice and sophistication . Don’t brush off these essays and give minimal effort, they want you to pass.

Writing the Essay

Your introductory paragraph should realistically comprise of your thesis and introduce your response to the prompt. Your introduction can be just one sentence with your thesis, or you can build context by prefacing your argument or claim with things you learned from the sources. Avoid using “I”. 

Your body paragraphs should be where you spend most of your time writing. Remember what the rubric says about relationships and connections between the sources. Look for key similarities and differences that may lend you to choose a main idea from the set. They all have something in common!

After you have an idea of your main points, start with a topic sentence that is essentially a thesis for the paragraph. Explain what you’re going to discuss and how it relates back to the prompt (or broader context, if applicable).

After introducing your topic sentence , begin using your evidence and elaborating in complete, complex sentences. If you planned your essay well enough, you may even be able to just copy what you have written down and just spend time elaborating on the sources. This maximizes your time and gives you some space to develop an even more complex argument . 2-3 sentences of elaboration is the sweet spot if you cover all your bases.

After you’ve done the steps above, do the same for the next body paragraph.

Once you reach your conclusion , state for the final time your thesis and the points you mentioned in your body paragraphs . Someone should be able to read your conclusion and get a good idea of what it is you discussed in your response, so make it informative and a good representation of your work!

And once you’ve reached this point, you’re all done! Give your essay a read and fix any mechanical or grammatical issues that you may stumble upon. After that, move on to the next essay and keep your head high-- you’re one step closer to finishing the exam! ✋

Fiveable

Student Wellness

Stay connected.

© 2024 Fiveable Inc. All rights reserved.

How to Write a Synthesis Essay AP Lang | Guide to Getting a Perfect Score

The AP Language and Composition exam is notoriously difficult. Even top, A+ language arts students have found it exceptionally challenging. 

the ap language and composition exam is notoriously difficult

Beyond diligently studying with a  top-rated AP Lang review book , you should also carefully review our comprehensive guide on how to write a synthesis essay for AP Lang featured in this article.

Once you complete the supremely humbling hour-long multiple-choice section, making up only 45% of your score, the trial has only just begun. Now, already fatigued from the first section, you must begin the grueling, two-hour free response section of the exam, which accounts for a whopping 55% of your final exam score. This section requires you to compose three essays of distinct types: an argumentative essay, a rhetorical analysis essay ( you also need to learn how to use AP Lang rhetorical devices ) and, the most dreaded of all, the synthesis essay.

Luckily, there are some great options to help you prepare to nail your synthesis essay. So, read on for everything you need to know to knock your synthesis essay out of the park!

How to Write Synthesis Essays AP Lang

One of the most challenging aspects of the AP synthesis essay is figuring out what the prompt is asking you to do. After all, what does it mean to “synthesize” something anyway? And what’s the difference between a synthesis essay and an evidence-based argumentative essay?

In this guide, we’ll answer these questions and a lot more. We’ll help you establish a solid understanding of what synthesis is, how to do it, and how to use it to write a high-scoring essay, according to College Board’s AP Language and Composition scoring guidelines . Lastly, we’ll leave you with some advice about things to be sure and attend to in your essay, as well as the most important things to avoid. 

Introduction to Syntheses

Before we get into all the details and advice on how to write a quality synthesis essay, we first need to answer a critical question: what is a synthesis essay?

Simply put, a synthesis essay is a piece of writing that brings together information and ideas from two or more sources. 

The synthesis part comes in as you begin to develop connections between the sources

The synthesis part comes in as you begin to develop connections between the sources, whether they are in agreement, disagreement, approach the same topic from different angles, or simply provide ideas on different topics that can in some way contribute some other discourse.

This, of course, is just a very basic introduction to what a synthesis essay is.

Throughout this article, the concept will surely become clearer to you.

Two Types of Syntheses

There are essentially two types of synthesis essay that you will run into: argumentative syntheses and explanatory syntheses. Luckily, you really only need to master one of them, the argumentative synthesis, as this is the most common expectation on AP exams.

Explanatory Synthesis

An explanatory synthesis essay is exactly what it sounds like. It the type of writing in which you will be asked to explain the arguments and information presented in your sources. You should also seek out connections and contrasting elements between the sources in order to give your essay a certain level of nuance and to display your deeper understanding and reasoning skills. 

Most of the time, AP Language and Composition exams won’t focus on explanatory synthesis essays. However, it would be wise to use some explanatory techniques even within your argumentative synthesis essay.

Argumentative Synthesis

When someone asks, “what is a synthesis in writing?” they’re typically referring to argumentative synthesis, and this is especially the case for the AP exam. 

At its most basic, an argumentative synthesis essay is on in which you must present your own opinions and support them with appropriate ideas and information from your sources. Most importantly, the thesis for your argumentative synthesis essay must be a proposition that can be debated. That is, there must be another potential argument against your own. 

Standards for Synthesis Essays

unfortunately, these prompt-specific rubrics are not available to the public until after the exams

The people who will be scoring your AP Lang synthesis essay use a very clearly defined rubric to determine your score according to various criteria. Unfortunately, these prompt-specific rubrics are not available to the public until after the exams.

That said, we can still gain a wealth of useful information from past scoring guidelines.

Here are some of the elements of high-scoring essays that seem to be fairly constant from exam to exam, regardless of the essay prompt topic:

Supply useful context on the topic

Give a sense of why the topic is important

Engage with the complexity of the subject

Foreground your opinion on the topic

Offer thorough and thoughtful analysis of quotations, paraphrases, etc.

Synthesize source material by finding connections with your own ideas and opinions

Properly attributes ideas to sources

Conclude with more than just a summary by answering the “so what?” question

Techniques for Developing Synthesis Essays

Before you can start writing a quality synthesis essay, you need to spend some time developing your ideas and seeing how they do or do not relate to your source materials.

The following is a list of steps that you should always take before you start writing the bulk of your actual essay. These guidelines will be enormously helpful when it comes time to figure out what you want to say in your essay. If you read this carefully and take seriously these suggestions, you’ll have no trouble coming up with interesting and complex ideas for your essay.

What’s your purpose? Before you do anything, you need to determine what the prompt is asking you to do. Obviously, it’s going to ask you to synthesize some stuff, but keep an eye out for these helpful guide words:

Compare/Contrast

Read the source material; then, read it again to annotate. Once you’ve read through the sources once, go back and reach each one again, this time with a pencil to underline and add notes as you go.

Formulate your thesis statement. After reading and taking notes on your sources, you’re ready to brainstorm your thesis statement. As you do this, try to keep track of potential aspects of each source that you can use to support your claim. 

Sketch an outline. When you have a thesis statement down (at least a tentative one—you should always be open to revising it as you go), you should sketch a simple outline that includes your thesis statement, supporting points that you can use for topic sentences, and a rough idea of how you will incorporate your sources. 

If you follow these steps closely, there’s no doubt that you’ll be ready to start writing what is sure to be a clever, thoughtful, and nuanced synthesis essay.

How to Write a Good Synthesis Essay?

as you go about the work of composing your essay, there are several approaches and strategies to can implement at different points in the essay

Now that we know what a synthesis essay is, what AP exam scorers will be looking for in your essay, and what techniques you can use to develop your essay topic, it’s time to look at some different strategies for how to actually write a synthesis essay in AP Lang.

As you go about the work of composing your essay, there are several approaches and strategies to can implement at different points in the essay.

None of these elements will be particularly effective in isolation, so be sure to use a variety of these strategies to enhance the complexity and depth of your argument.

Summarize. Summarizing ideas and source material is easy, and for that reason it is not going to do a whole lot in the way of getting you a great score. It is, however, an incredibly useful tool that, when used in conjunction with other strategies on this list, can be quite effective.

Compare and Contrast. This is typically the level of synthesis that most people start at after summarizing the content they are considering. It’s not exactly “high-order” analysis, but it can be very useful in establishing the positions of your sources and creating a foundation on which you can present your own ideas and opinions.

Give an Example. A great way to show you understand a certain concept is to apply your understanding through an example. You can think of examples that illustrate the concepts you’re dealing with as a way to clarify your topic and also to support your own arguments. 

“They Say, I Say.” This is the classic, and possibly most effective, synthesis move. After you’ve presented a quotation or paraphrase of one or more sources’ ideas, move on to explain your own position as it relates to theirs. It becomes much easier to state your own ideas and opinions when you do it within the context of the larger discussion of the topic.

Synthesis Essay Structure

One of the most sure-fire ways to earn a passing score on your synthesis essay is to ensure you structure it effectively. To do this well, it’s a smart idea, prior to writing your essay, to sketch out a quick outline of the essay’s structure. Your outline doesn’t need to be especially detailed, but it will be tremendously helpful for you to have a general plan to work from.

Another good idea that will help you learn how to structure your essay is by looking at a synthesis example.

Helpfully, the College Board’s AP Language and Composition exam site supplies several samp le essays for you to get an idea of what good, average, and poor synthesis essays look like.

if you take some time to read these examples, you’ll have a much better vision of what the scorers are looking for in terms of a well-structured synthesis essay

If you take some time to read these examples, you’ll have a much better vision of what the scorers are looking for in terms of a well-structured synthesis essay.

That said, there is a very basic structure for synthesis essays that, if followed carefully, will guarantee that your essay is, at the very least, structured logically. This format is essentially the standard format for any basic five-paragraph essay. Take a look.

Basic Essay Structure

Introduction Paragraph

Give some BACKGROUND information and CONTEXT on the general topic of your essay.

Briefly introduce the SOURCES that you will be using.

Present your THESIS STATEMENT (this is usually the very last sentence of the paragraph).

Body Paragraph

Start with a TOPIC SENTENCE that supports your thesis statement.

Cite one or more SOURCES that support your topic sentence.

Provide COMMENTARY and ANALYSIS on the ideas you have just cited from your sources. 

Use a TRANSITION WORD or PHRASE to guide the reader logically toward the ideas you will present in the topic sentence of the next paragraph.

Body Paragraphs 2-?

Repeat the steps in the previous section with different topic sentences and source references, all of which must support your thesis statement. 

Include as many body paragraphs as you have points and topic sentences to justify. Typically, a good AP Lang synthesis essay will have around 3-4 well-constructed and reasoned body paragraphs, but this is just a general guideline.

Conclusion Paragraph

Restate your THESIS statement in a new and interesting way (that is, do not simply repeat your thesis word-for-word as it appears in the introduction!).

Tie it all together by briefly summarizing your main points.

Answer the “SO WHAT?” QUESTION by explaining why your argument matters and what the implications of it might be. Try to broaden your scope in order to show the reader how it fits in with the “big picture.”

Synthesis Writing Dos and Don’ts

ap lang synthesis essay time

Finally, we want to leave you with a quick list of things to strive for and things to avoid at all costs. 

DO Develop a Strong, Clear Thesis Statement

DO Use Topic Sentences 

DO Cite Your Sources Accurately and Appropriately

DO Sketch a Basic Outline 

DO Pace Yourself

DO Proofread and Revise Your Essay Carefully

DON’T Overdo It with Summaries

DON’T Start Paragraphs with Quotations

DON’T Get Overwhelmed by the Sources

DON’T Use Other People’s Ideas without Citing Your Sources

DON’T Use Overly Lengthy Quotations or Paraphrases

Leonard Haggin

I created this site to help students like you learn from the experiences my team had learned during our extensive academic careers. I am now studying Law at Stanford, but I also make time to write articles here in order to help all you fellow students advance in your academic careers and beyond. I hope our efforts on Study Prep Lounge will arm you with the knowledge you need to overcome whatever trial or test you find in front of you.

Leave a Reply:

Save my name, email, and website in this browser for the next time I comment.

Automated page speed optimizations for fast site performance

logo-type-white

AP® English Language

How to ace the ap® english language and composition synthesis essay.

  • The Albert Team
  • Last Updated On: March 1, 2022

how_to_ace_the_ap_english english language synthesis essay

The newest section of the AP® English Language and Composition Exam, the synthesis essay, is one of three essays you will be completing during the examination’s 2-hour free-response period. However, you’ll also have a 15-minute reading and planning period just for this essay, and if you use this time to plan effectively, you can’t go wrong.

Before we get into specific advice on how to handle the AP® English Language and Composition synthesis essay, you need to know what this part of the test really is. It is very similar to the argumentative essay you will also write as part of this exam, except that you are provided with a wealth of source material from which to draw some support for your ideas.

While this in some ways makes the AP® English Language and Composition synthesis essay easier than the argument essay (because you can use quotations, point to authoritative sources for support, etc.), there is an extra element of complexity, and the AP® readers want to see how well you can sort through your source material and put it to good use – which makes planning all that much more important. This brings us to our first tip…

1. Use Your 15-Minute Planning Period Wisely.

The main purpose of this 15-minute period is to give you time to read the source materials. This essay will present you with several sources providing different information about or opinions on a certain topic. Make sure you don’t just skim them, but read them closely – make notes, underline key sections you may want to quote later, etc.

You should also begin outlining your essay and considering your opinion on the subject; have this opinion in mind before you start writing the essay, as you will use it to construct your thesis.

You’ve already learned how to structure persuasive essays in this class and in other classes you have taken; put that knowledge to good use now, and have your main points set out before you start writing. Try to have a thesis statement written by the time you start the essay – your thesis should establish your opinion and the general reasons you feel this way; the rest of your essay will go on to justify and exemplify these reasons. Also write down some of the main points upon which you will base subsequent paragraphs and mark quotes or sections of the sources you can use in each of these paragraphs.

2. Evaluate Your Sources.

ap exams score

Every source you can use for the AP® Language and Composition synthesis essay will have a small box above it explaining where it comes from and who said it – to see exactly what this looks like, check out the free synthesis essay sample questions at AP® Central. There are also public sample questions available there for the rest of the AP® English and Composition Exam .

Keep all information about your sources in mind when you’re quoting them or using them to support your arguments. What journal an article appeared in can say a great deal about its potential biases. For example, consider a question on the environmental impacts of corporate practices – an environmental journal is obviously going to be biased in favor of more environmental regulation, while a report from a company spokesperson will probably gloss over some of the negative impacts of his company. Think critically.

3. Keep Your Tone Consistent.

There is no hard-and-fast advice about what tone you should take – some students try to inject a little humor into their essays while others prefer to be as serious as possible, some are extremely critical and others more accepting. However, the one thing you really have to do while writing the AP® Language and Composition synthesis essay (or any other essay) is keep your tone consistent. Jot some tone-related ideas down as you outline during the 15-minute reading period, and keep in mind everything you’ve learned about tone and other aspects of rhetoric so far this year.

4. Use Rhetorical Technique to Your Advantage!

The various rhetorical practices you’ve been learning about all year can be put to good use here. This class and this test aren’t just about recognizing and analyzing these techniques when others use them, but about preparing you for college and your career by teaching you how to use them effectively yourself. However, this isn’t just about writing a beautiful essay, so read on to Tip # 5!

5. Your Argument Must be Well-Crafted.

The AP® English Language and Composition Exam synthesis essay does not have right or wrong answers; rather, it asks you for your opinion. The AP® Examiner cannot take points off because she disagrees with you. However, you must show logical basis for your opinion, drawing on both the sources AND your own knowledge and experience.

To do this, make sure you have a clear and complete thesis. Make sure the ideas expressed in the beginning of each paragraph or section support the thesis, and that you in turn show how those ideas are supported by a source or through your own knowledge and experience. Don’t generalize or write anything down that you can’t support.

Looking for AP® English Language practice?

Kickstart your AP® English Language prep with Albert. Start your AP® exam prep today .

Interested in a school license?​

Popular posts.

AP® Physics I score calculator

AP® Score Calculators

Simulate how different MCQ and FRQ scores translate into AP® scores

ap lang synthesis essay time

AP® Review Guides

The ultimate review guides for AP® subjects to help you plan and structure your prep.

ap lang synthesis essay time

Core Subject Review Guides

Review the most important topics in Physics and Algebra 1 .

ap lang synthesis essay time

SAT® Score Calculator

See how scores on each section impacts your overall SAT® score

ap lang synthesis essay time

ACT® Score Calculator

See how scores on each section impacts your overall ACT® score

ap lang synthesis essay time

Grammar Review Hub

Comprehensive review of grammar skills

ap lang synthesis essay time

AP® Posters

Download updated posters summarizing the main topics and structure for each AP® exam.

Interested in a school license?

ap lang synthesis essay time

Bring Albert to your school and empower all teachers with the world's best question bank for: ➜ SAT® & ACT® ➜ AP® ➜ ELA, Math, Science, & Social Studies aligned to state standards ➜ State assessments Options for teachers, schools, and districts.

Get in touch with us

Are you sure you want to logout?

Study abroad.

bannerAd

Steps to Draft AP Lang Synthesis Essay

The synthesis essay AP lang is part of the AP language and composition exam . It is one of the courses in the juniors’ curriculum, making it compulsory for them to appear for the exam. Judging the argumentation skills with the ability to establish a line of reasoning, answering the exam requires students to follow a pattern. Easily cracked with the practice, here is how you should go on to learn to write the perfect answer. 

What Does the AP Language and Composition Exam Look Like?

The AP Language and Composition Exam is taken in the duration of 3 hours and 15 minutes and comprises two sections. 

parallel

  • The first section is multiple choice questions where the reading and writing abilities of the students are assessed through a total of 45 questions. The section holds 45% weightage and must be completed in an hour. 
  • The second section is free-response questions, where there are three questions holding 55% weightage. The time offered here is 2 hours and 45 minutes, where 15 minutes are allotted to the reading period. There are three essay questions here, with 40 minutes allotted to each. The first question is the synthesis AP lang essay , the second is the rhetorical analysis, and the third is an argument. 

How is the AP Lang Synthesis Essay Question Asked?

The first free-response question puts forward a specific topic and six to seven sources of information. The students are offered a variety of information where two are visuals, and at least one is the quantitative source. The remaining sources are excerpts from the text, and each of these sources will be around 500 words. The candidates are expected to synthesise their essays and reflect their viewpoints based on and backed by at least three provided sources. 

What Are the Factors or Criteria of the Assessment of the Synthesis Essay?

The synthesis essay is assessed by the invigilator considering the following points: 

parallel

  • The presence of a thesis statement in response to the prompt could start a series of argument
  • Utilisation of at least three sources to support the evidence. The sources must be clearly cited or indicated in parentheses
  • Ability to relate the evidence and thesis 
  • Capability to exhibit an understanding of the rhetorical situation 
  • Usage of proper grammar and its rules in the essay 

How To Write a Synthesis Essay AP Lang?

This is the prime question when dealing with the synthesis essay AP Lang . Let’s begin by breaking it down into smaller and more understandable components: 

Step 1: Prompt Analysis

Since you have only a completely new question and sources in hand, the first step will be to analyse the available information. Re-read the prompt at least twice or thrice and identify and note down the prime fact each prompt is conveying. For instance, the first para will be an introduction making you familiar with the topic and the second will offer insights and help you build a view or take a specific side. The third paragraph can be instructions on what is expected in the essay. 

parallel

Step 2: Choose the Source

For proper choice usage, you must know what each source encompasses. It leads to the recommendation of going through each source. Reading the source is recommended to ensure you know which sources back your side and which don’t. Summarise this information on each source with a word about which side they are on. Glancing over the sources will also aid in deciding the utilisation of sources in the topic and is efficient compared to randomly choosing the sources without reading. It needs to be done during the reading time and is aimed at saving the essay writing time. 

Step 3: Begin with the Thesis Statement

You need to begin by conveying your side or stand and must establish a line of reasoning for scoring good marks. It should be strictly related to the topic provided and must not wander about the general situations. The thesis statement needs to be clearly mentioned and should not be a copy of the complete or part of the prompt. The reader must understand at the beginning what the essay will be about or what they should expect, along with the reason for your thoughts in your write-up. 

parallel

Step 4: Structure Your Essay

An essay is supposed to be sequential. Structure with an outline is the only method to do it. This will not go on your fair copy but will be a guide on how you will approach writing the essay. You are expected to divide the essay into an introduction, body, and conclusion. The introduction will carry a brief context of the topic and sources, along with communication about your stand or side on the topic. 

The body can be divided into three paragraphs. Each paragraph will begin with supporting or opposing information, followed by the evidence and your thoughts on the same. The conclusion will be an overview or summarisation of the reasoning presented in your essay and a final line proving your point. 

parallel

Step 5: Fill the Outline With the Essay

You have the structure of what exactly you will be covering in the essay. You know how it has to go, and now the only thing left is writing the essay. Fill up your structure like the blanks. You will be communicating and reflecting your mind and thoughts to the invigilator with the essay. Ensure to depict how the evidence takes or opposes the side you are on. You can get help by incorporating quotes from the source. Overall, you will be explaining the source and clearly indicating the sources used within the parenthesis or by mention in the essay, as it is the criteria determining your marks. 

What Are the Do’s and Don’ts While Writing a Synthesis Essay AP Lang Exam?

Here are some important do’s and don’ts to follow: 

parallel

  • Don’t limit your source usage to minimum requirements. Use at least four of them.
  • Don’t end up plagiarizing by summarisation or copying the question part. 
  • Do use your freedom to agree or disagree. 
  • Do go through all the sources for better choices. 
  • Do use quotes to enhance readability 
  • Do cite the sources used 
  • Do write the thesis statement 
  • Do write the notes when going through the topic and sources 

Conclusion 

The AP lang synthesis question is a part of the second section carrying more weightage in the exam. Being one of the three free-response questions, the candidates are expected to put forward their perspective and stand while tracking back their reasoning to the sources. Going through the complete prompt and all the sources tends to assist in scoring. Further, writing assistance is available on creating a structure to fill it up with your essay. 

Frequently Asked Questions

What is the maximum possible score in the synthesis part.

The synthesis part is of 6 points, where a maximum of 1 mark is awarded on thesis development, a maximum of 4 is awarded on evidence and commentary, and a maximum of 1 mark is awarded on the sophistication of thought.

parallel

Can I score well in the AP Language and Composition exam?

Yes, you can score well. However, it necessarily requires practice, a broad way of thinking, and properly deciding the flow of ideas in the essay.

How many questions are there for reading and writing?

There are 23 to 25 questions for reading and 20 to 22 questions for writing part in section I.

AP Lang Synthesis Essay

Relevant Articles

AP Calculus AB Exam

Strategies for Success: A Complete Overview of the AP Calculus AB Exam

The AP Calculus AB Exam serves as an important element …

Strategies for Success: A Complete Overview of the AP Calculus AB Exam Read More »

Digital AP Exams

Preparing for Success: A Guide to the Features of Digital AP Exams

Digital  Advanced Placement (AP) exams bring about a huge change …

Preparing for Success: A Guide to the Features of Digital AP Exams Read More »

Tips on Self-Study for AP Exams

How to Self-Study for AP Exams – 7 Steps Plan

Is it possible to crack the AP exam with self-study? …

How to Self-Study for AP Exams – 7 Steps Plan Read More »

card img

With Turito Study Abroad

card img

Get an Expert Advice from Turito

card img

With Turito CAP.

card img

With Turito Coding.

card img

With Turito RoboNinja

card img

1-on-1 tutoring for the undivided attention

How to Write the AP Lang Argument Essay (With Example)

December 14, 2023

ap lang argument essay example

We’d like to let you in on a little secret: no one, including us, enjoys writing timed essays. But a little practice goes a long way. If you want to head into your AP English Exam with a cool head, you’ll want to know what you’re getting into ahead of time. We can’t promise the AP Lang Argument Essay will ever feel like an island vacation, but we do have tons of hand tips and tricks (plus a sample essay!) below to help you do your best. This article will cover: 1) What is the AP Lang Argumentative Essay? 2) AP Lang Argument Rubric 3) AP Lang Argument Sample Prompt 4) AP Lang Argument Essay Example 5) AP Lang Argument Essay Example: Answer Breakdown.

What is the AP Lang Argument Essay?

The AP Lang Argument Essay is one of three essays included in the written portion of the AP English Exam. The full AP English Exam is 3 hours and 15 minutes long, with the first 60 minutes dedicated to multiple-choice questions. Once you complete the multiple-choice section, you move on to three equally weighted essays that ask you to synthesize, analyze, and interpret texts and develop well-reasoned arguments. The three essays include:

Synthesis essay: You’ll review various pieces of evidence and then write an essay that synthesizes (aka combines and interprets) the evidence and presents a clear argument. Read our write-up on How to Write the AP Lang Synthesis Essay here.

Argumentative essay: You’ll take a stance on a specific topic and argue your case.

Rhetorical essay: You’ll read a provided passage, then analyze the author’s rhetorical choices and develop an argument that explains why the author made those rhetorical choices. Read our write-up on How to Write the AP Lang Rhetorical Essay here.

AP Lang Argument Essay Rubric

The AP Lang Argument Essay is graded on 3 rubric categories : Thesis, Evidence and Commentary, and Sophistication . How can you make sure you cover all three bases in your essay? We’ll break down each rubric category with dos and don’ts below:

  • Thesis (0-1 point)

When it comes to grading your thesis, AP Exam graders are checking off a box: you either have a clear thesis or you don’t. So, what crucial components of a thesis will get you your check mark?

  • Make sure your thesis argues something . To satisfy your graders, your thesis needs to take a clear stance on the issue at hand.
  • Include your thesis statement in your intro paragraph. The AP Lang Argumentative essay is just that: an essay that makes an argument, so make sure you present your argument right away at the end of your first paragraph.
  • A good test to see if you have a thesis that makes an argument for your AP Lang Argumentative Essay: In your head, add the phrase “I agree/disagree that…” to the beginning of your thesis. If what follows doesn’t logically flow after that phrase (aka if what follows isn’t an agreement or disagreement), it’s likely you’re not making an argument.
  • In your thesis, outline the evidence you’ll cover in your body paragraphs.

AP Lang Argument Essay Rubric (Continued)

  • Avoid a thesis that merely restates the prompt.
  • Avoid a thesis that summarizes the text but does not make an argument.
  • Avoid a thesis that weighs the pros and cons of an issue. Your job in your thesis is to pick a side and stick with it.
  • Evidence and Commentary (0-4 points)

This rubric category is graded on a scale of 0-4 where 4 is the highest grade. Unlike the rhetorical and synthesis essays, the evidence you need to write your AP Lang Argument Essay is not provided to you. Rather, you’ll need to generate your own evidence and comment upon it.

What counts as evidence?

Typically, the AP Lang Argument Essay prompt asks you to reflect on a broad cultural, moral, or social issue that is open to debate. For evidence, you won’t be asked to memorize and cite statistics or facts. Rather, you’ll want to bring in real-world examples of:

  • Historical events
  • Current-day events from the news
  • Personal anecdotes

For this essay, your graders know that you’re not able to do research to find the perfect evidence. What’s most important is that you find evidence that logically supports your argument.

What is commentary?

In this essay, it’s important to do more than just provide examples relevant evidence. After each piece of evidence you include, you’ll need to explain why it’s significant and how it connects to your main argument. The analysis you include after your evidence is commentary .

  • Take a minute to brainstorm evidence that logically supports your argument. If you have to go out of your way to find the connection, it’s better to think of different evidence.
  • Include multiple pieces of evidence. There is no magic number, but do make sure you incorporate more than a couple pieces of evidence that support your argument.
  • Make sure you include more than one example of evidence, too. Let’s say you’re working on an essay that argues that people are always stronger together than apart. You’ve already included an example from history: during the civil rights era, protestors staged group sit-ins as a powerful form of peaceful protest. That’s just one example, and it’s hard to make a credible argument with just one piece of evidence. To fix that issue, think of additional examples from history, current events, or personal experience that are not related to the civil rights era.
  • After you include each piece of evidence, explain why it’s significant and how it connects to your main argument.
  • Don’t summarize or speak generally about the topic. Everything you write must be backed up with specific and relevant evidence and examples.
  • Don’t let quotes speak for themselves. After every piece of evidence you include, make sure to explain and connect the evidence to your overarching argument.

AP Lang Argument Essay (Continued)

  • Sophistication (0-1 point)

According to the College Board , one point can be awarded to AP Lang Argument essays that achieve a high level of sophistication. You can accomplish that in four ways:

  • Crafting a nuanced argument by consistently identifying and exploring complexities or tensions.
  • Articulating the implications or limitations of an argument by situating it within a broader context.
  • Making effective rhetorical choices that consistently strengthen the force and impact of the student’s argument.
  • Employing a style that is consistently vivid and persuasive.

In sum, this means you can earn an additional point for going above and beyond in depth, complexity of thought, or by writing an especially persuasive, clear, and well-structured essay. In order to earn this point, you’ll first need to do a good job with the fundamentals: your thesis, evidence, and commentary. Then, to earn your sophistication point, follow these tips:

  • Outline your essay before you begin to ensure it flows in a clear and cohesive way.
  • Include well-rounded evidence. Don’t rely entirely on personal anecdotes, for example. Incorporate examples from current events or history, as well.
  • Thoroughly explain how each piece of evidence connects to your thesis in order to fully develop your argument.
  • Explore broader implications. If what you’re arguing is true, what does that mean to us today? Who is impacted by this issue? What real-world issues are relevant to this core issue?
  • Briefly explore the other side of the issue. Are the instances where your argument might not be true? Acknowledge the other side, then return to proving your original argument.
  • Steer clear of generalizations (avoid words like “always” and “everyone”).
  • Don’t choose an argument you can’t back up with relevant examples.
  • Avoid complex sentences and fancy vocabulary words unless you use them often. Long, clunky sentences with imprecisely used words are hard to follow.

AP Lang Argument Sample Prompt

The sample prompt below is published online by the College Board and is a real example from the 2021 AP English Exam. The prompt provides background context, essay instructions, and the text you need to analyze.

Suggested time—40 minutes.

Many people spend long hours trying to achieve perfection in their personal or professional lives. Similarly, people often demand perfection from others, creating expectations that may be challenging to live up to. In contrast, some people think perfection is not attainable or desirable.

Write an essay that argues your position on the value of striving for perfection.

In your response you should do the following:

  • Respond to the prompt with a thesis that presents a defensible position.
  • Provide evidence to support your line of reasoning.
  • Explain how the evidence supports your line of reasoning.
  • Use appropriate grammar and punctuation in communicating your argument.

AP Lang Argument Essay Example

As the old phrase says, “Practice makes perfect.” But is perfection something that is actually attainable? Sometimes, pushing for perfection helps us achieve great things, but most often, perfectionism puts too much pressure on us and prevents us from knowing when we have done the best we can. Striving for perfection can only lead us to shortchange ourselves. Instead, we should value learning, growth, and creativity and not worry whether we are first or fifth best.

Students often feel the need to be perfect in their classes, and this can cause students to struggle or stop making an effort in class. In elementary and middle school, for example, I was very nervous about public speaking. When I had to give a speech, my voice would shake, and I would turn very red. My teachers always told me “relax!” and I got Bs on Cs on my speeches. As a result, I put more pressure on myself to do well, spending extra time making my speeches perfect and rehearsing late at night at home. But this pressure only made me more nervous, and I started getting stomach aches before speaking in public.

Once I got to high school, however, I started doing YouTube make-up tutorials with a friend. We made videos just for fun, and laughed when we made mistakes or said something silly. Only then, when I wasn’t striving to be perfect, did I get more comfortable with public speaking.

AP Lang Argumentative Essay Example (Continued)

In the world of art and business and science, perfectionism can also limit what we are able to achieve. Artists, for example, have to take risks and leave room for creativity. If artists strive for perfection, then they won’t be willing to fail at new experiments and their work will be less innovative and interesting. In business and science, many products, like penicillin for example, were discovered by accident. If the scientist who discovered penicillin mold growing on his petri dishes had gotten angry at his mistake and thrown the dishes away, he would never have discovered a medicine that is vital to us today.

Some fields do need to value perfection. We wouldn’t like it, for example, if our surgeon wasn’t striving for perfection during our operation. However, for most of us, perfectionism can limit our potential for learning and growth. Instead of trying to be perfect, we should strive to learn, innovate, and do our personal best.

AP Lang Argument Essay Example: Answer Breakdown

The sample AP Lang Argumentative Essay above has some strengths and some weaknesses. Overall, we would give this essay a 3 or a 4. Let’s break down what’s working and what could be improved:

  • The essay offers a thesis that makes a clear argument that is relevant to the prompt: “Striving for perfection can only lead us to shortchange ourselves. Instead, we should value learning, growth, and creativity and not worry whether we are first or fifth best.”
  • The first body paragraph provides evidence that supports the essay’s thesis. This student’s personal anecdote offers an example of a time when perfectionism led them to shortchange themselves.
  • The second body paragraph provides additional evidence that supports the essay’s thesis. The example describing the discovery of penicillin offers another example of a situation in which perfectionism might have limited scientific progress.
  • The writer offers commentary explaining how her examples of public speaking and penicillin illustrate that we should “value learning, growth, and creativity” over perfectionism.
  • The essay follows one line of reasoning and does not stray into tangents.
  • The essay is organized well with intro, body, and concluding paragraphs. Overall, it is easy to read and is free of grammar errors.

What could be improved:

  • Although the second body paragraph provides one good specific example about the discovery of penicillin, the other examples it offers about art and business are only discussed generally and aren’t backed up with evidence. This paragraph would be stronger if it provided more examples. Or, if this writer couldn’t think of examples, they could have left out mentions of art and business altogether and included alternate evidence instead.
  • This writer would more thoroughly support their argument if they were able to offer one more example of evidence. They could provide another personal anecdote, an example from history, or an example from current events.
  • The writer briefly mentions the other side of the argument in their concluding paragraph: “Some fields do need to value perfection. We wouldn’t like it, for example, if our surgeon wasn’t striving for perfection during our operation.” Since it’s so brief a mention of the other side, it undermines the writer’s overall argument. This writer should either dedicate more time to reflecting on why even surgeons should “value learning, growth, and creativity” over perfectionism, or they should leave these sentences out.

AP Lang Argument Essay Example—More Resources

Looking for more tips to help you master your AP Lang Argumentative Essay? Brush up on 20 Rhetorical Devices High School Students Should Know and read our Tips for Improving Reading Comprehension .

If you’re ready to start studying for another part of the AP English Exam, find more expert tips in our How to Write the AP Lang Synthesis and How to Write the AP Lang Rhetorical Essay blog posts.

  • High School Success

' src=

Christina Wood

Christina Wood holds a BA in Literature & Writing from UC San Diego, an MFA in Creative Writing from Washington University in St. Louis, and is currently a Doctoral Candidate in English at the University of Georgia, where she teaches creative writing and first-year composition courses. Christina has published fiction and nonfiction in numerous publications, including The Paris Review , McSweeney’s , Granta , Virginia Quarterly Review , The Sewanee Review , Mississippi Review , and Puerto del Sol , among others. Her story “The Astronaut” won the 2018 Shirley Jackson Award for short fiction and received a “Distinguished Stories” mention in the 2019 Best American Short Stories anthology.

  • 2-Year Colleges
  • Application Strategies
  • Big Picture
  • Career & Personality Assessment
  • College Essay
  • College Search/Knowledge
  • College Success
  • Costs & Financial Aid
  • Extracurricular Activities
  • Graduate School Admissions
  • High Schools
  • Law School Admissions
  • Medical School Admissions
  • Navigating the Admissions Process
  • Online Learning
  • Summer Program Spotlight
  • Summer Programs
  • Test Prep Provider Spotlight

College Transitions Sidebar Block Image

“Innovative and invaluable…use this book as your college lifeline.”

— Lynn O'Shaughnessy

Nationally Recognized College Expert

College Planning in Your Inbox

Join our information-packed monthly newsletter.

Sign Up Now

IMAGES

  1. AmStud

    ap lang synthesis essay time

  2. How to Write an AP Lang Synthesis Essay: A Complete Guide

    ap lang synthesis essay time

  3. How to Write the AP Lang Synthesis Essay: Read the Prompt

    ap lang synthesis essay time

  4. AP Lang Synthesis Essay Notes by Emily Mendez

    ap lang synthesis essay time

  5. How to Write the AP Lang Synthesis Essay: Write a Thesis

    ap lang synthesis essay time

  6. How to Outline a Synthesis Essay for AP Lang

    ap lang synthesis essay time

VIDEO

  1. Synthesis Essay Writing Week! (Julius Caesar Unit Week 4) #highschoolteacher #highschoolenglishteach

  2. Synthesis essay PROCESS

  3. AP Lang Course Description Video

  4. importance of Time || Value of Time#valueoftime#shorts

  5. Do I Need to Read All the Synthesis Sources? #aplang

  6. Synthesis Essay Conclusion Tips

COMMENTS

  1. Synthesis Essay Materials

    Sample 2 An additional sample synthesis essay question is provided here. AP English Language and Composition Synthesis Essay #2 (.pdf/338KB) The two synthesis essay questions below are examples of the question type that has been one of the three free-response questions on the AP English Language and Composition Exam as of the May 2007 exam.

  2. How to Write a Perfect Synthesis Essay for the AP Language Exam

    This hour consists of a recommended 15 minute reading period and a 40 minute writing period. Keep in mind that these time allotments are merely recommendations, and that exam takers can parse out the allotted 60 minutes to complete the synthesis essay however they choose. Now, here's what the structure of the AP Lang synthesis essay looks like.

  3. How to Write the AP Lang Synthesis Essay + Example

    It's recommended that students spend an hour on this essay—15 minute reading period, 40 minutes writing, and 5 minutes of spare time to check over work. Each synthesis essay has a topic that all the sources will relate to.

  4. Complete Guide on AP Lang Synthesis Essay

    Written By William James Last Updated: 01 November, 2023 The last component of your basic college English composition course is the AP Lang test. The most crucial—and sometimes challenging—component is the AP Language Synthesis Essay. It's the cornerstone of your future writing abilities, but if you're not prepared, it can be challenging.

  5. PDF AP English Language and Composition

    AP® English Language and Composition 2022 Scoring Guidelines . Synthesis Essay 6 points . Since the early 2000s, the United States government and a number of corporations have sponsored initiatives to improve education in the STEM disciplines: science, technology, engineering, and mathematics.

  6. How to Write the AP Lang Synthesis Essay with Example

    Most recently, her work has been featured in Belle Ombre, Flypaper Lit, and Mag 20/20, among others, and she was nominated for the Mary Troy Prize in Fiction. How to write the AP Lang synthesis essay with examples. The synthesis essay is the first free response essay on the AP Lang exam.

  7. Tackling the AP English Language and Composition essays: part 1

    CC | Tackling the AP English Language and Composition essays: part 1. More than any other test, the AP English Language and Composition Exam is dominated by essays. Three timed essays—the Synthesis Essay, Rhetoric Essay, and Argument Essay—will take up most of your time on the exam, and count for more than fifty percent of your score.

  8. PDF AP English Language and Composition

    Question 1 Note: Student samples are quoted verbatim and may contain spelling and grammatical errors. Overview The synthesis prompt for this year asked students to use material from six provided sources and develop a position on the place, if any, of handwriting instruction in today's schools.

  9. How to Write the AP Lang Synthesis Essay + Essay Template

    July 19, 2022 10 minutes Share the article AP Lang test is the logical conclusion to the introductory college English composition course. And its most important (and often difficult) part is the AP Lang synthesis essay. Despite it being the very basic layer of your future composition skills, it's a very complicated challenge to approach unprepared.

  10. Synthesis Overview

    10 min. (to read sources) + 5 min. (planning) + 35 min. (writing) = 50 min. How to Rock the Synthesis Question: The Rubric The synthesis question is scored on a six-point , and each point can be earned individually. This means that you can get points in one category, but not in others. It all depends on how well you accomplish each level on the .

  11. How to Write a Synthesis Essay AP Lang

    Typically, a good AP Lang synthesis essay will have around 3-4 well-constructed and reasoned body paragraphs, but this is just a general guideline. Conclusion Paragraph. Restate your THESIS statement in a new and interesting way (that is, do not simply repeat your thesis word-for-word as it appears in the introduction!).

  12. Acing the AP® English Language and Composition Synthesis Essay

    The newest section of the AP® English Language and Composition Exam, the synthesis essay, is one of three essays you will be completing during the examination's 2-hour free-response period. However, you'll also have a 15-minute reading and planning period just for this essay, and if you use this time to plan effectively, you can't go wrong.

  13. AP English Language Exam Practice: Synthesis Study Plan

    Resources you need to improve your Synthesis essay on the AP English Language and Composition exam. Includes revelant readings and practice problems. Note: For best results, click to highlight and copy/paste this list into your Fiveable Rooms Task Card to automatically create individual tasks. Jumpstart your studying in 5 seconds!

  14. Synthesis essay tips for AP Lang

    Hi! It's great that you're working on improving your synthesis essay skills in AP Lang. Here are some tips and advice for effectively using sources and organizing your essay: 1. Understand the prompt: Make sure you have a clear understanding of the prompt and what it's asking. This will help you formulate a strong thesis statement and determine which sources will be most relevant to support ...

  15. PDF AP English Language and Composition Free-Response Questions Scoring

    AP English Language Scoring Rubric, Free-Response Question 1-3 | SG 1 Scoring Rubric for Question 1: Synthesis Essay 6 points Reporting Category Scoring Criteria Row A Thesis (0-1 points) 4.B 0 points For any of the following: • There is no defensible thesis. • The intended thesis only restates the prompt.

  16. PDF AP English Language and Composition Synthesis Essay Sample Student

    AP® English Language and Composition Synthesis Essay Sample Student Responses The College Board: Connecting Students to College Success The College Board is a not-for-profit membership association whose mission is to connect students to college success and opportunity. Founded in 1900, the association is composed of more than 5,000 schools ...

  17. Cracking the AP Lang Synthesis Essay: Tips and Strategies

    The time offered here is 2 hours and 45 minutes, where 15 minutes are allotted to the reading period. There are three essay questions here, with 40 minutes allotted to each. The first question is the synthesis AP lang essay, the second is the rhetorical analysis, and the third is an argument. How is the AP Lang Synthesis Essay Question Asked?

  18. PDF ENGLISH LANGUAGE AND COMPOSITION

    2019 AP® ENGLISH LANGUAGE AND COMPOSITION FREE-RESPONSE QUESTIONS ENGLISH LANGUAGE AND COMPOSITION SECTION II Total time—2 hours and 15 minutes . Question 1 . Suggested reading and writing time—55 minutes. It is suggested that you spend 15 minutes reading the question, analyzing and evaluating the sources, and 40 minutes writing your response.

  19. AP English Language and Composition Exam

    Tue, May 14, 2024 8 AM Local AP English Language and Composition Exam Add to Calendar Exam Format The AP English Language and Composition Exam has question types and point values that stay consistent from year to year, so you and your students know what to expect on exam day. Section I: Multiple Choice 45 Questions | 1 hour | 45% of Exam Score

  20. Guide for AP Lang Synthesis Essay : r/APStudents

    Guide for AP Lang Synthesis Essay . I have seen a few posts with ppl struggling to write the essay, so here's how I did it. I'm not an English person, but this is how I would write question 1: An intro (Skip this if you're running out of time). Thesis statement including two of your sub-claims. I like a counterargument thesis, but a ...

  21. AP English Language and Composition Past Exam Questions

    Free-Response Questions. Download free-response questions from past exams along with scoring guidelines, sample responses from exam takers, and scoring distributions. If you are using assistive technology and need help accessing these PDFs in another format, contact Services for Students with Disabilities at 212-713-8333 or by email at ssd@info ...

  22. How to Write the AP Lang Argument Essay (With Example)

    Ap Lang Argumentative Essay - Expert advice on how to pen a winning essay + an AP Lang argument essay example to guide your writing.

  23. PDF AP English Language and Composition

    In your response you should do the following: Respond to the prompt with a thesis that presents a defensible position. Select and use evidence from at least three of the provided sources to support your line of reasoning. Indicate clearly the sources used through direct quotation, paraphrase, or summary.